Test Bank - Assurance Principles (Cpar)l

May 2, 2018 | Author: jsus22 | Category: Financial Audit, Internal Control, Audit, Internal Audit, Auditor's Report


Comments



Description

 Preface to the Philippine Standards on Quality Control, Auditing Review, Other Assuranceand Related Services  Philippine Framework for Assurance Engagements  Objective and General Principles governing an Audit of Financial Statements(PSA 200 <amended as a result of PSA 700 (revised)>) 1. The preface to the Philippine Standards on Quality Control, Auditing, Other Assurance and Related Services is issued to facilitate understanding of the I Objectives and operating procedures of the Auditing and Assurance Standards Council (AASC). II Scope and authority of the documents issued by the AASC. A. I only B. II only C. Both I and II D. Neither I nor II 2. Its mission is the promulgation of auditing standards, practices and procedures which shall be generally accepted by the accounting profession in the Philippines. A. Financial Reporting Standards Council (FRSC). B. Auditing and Assurance Standards Council (AASC). C. International Federation of Accountants (IFAC). D. Philippine Institute of Public Accountants (PICPA). 3. Which of the following are collectively referred to as “Engagement Standards”? A. PSAs and PSREs B. PSAs, PSREs and PSAEs C. PSAs, PSREs, PSAEs and PSRSs D. PSAs, PSREs, PSAEs, PSRSs and PSQCs 4. Which of the following Philippine Standards are to be applied to compilation engagements, engagements to apply agreed-upon procedures to information and other related services engagements as specified by the AASC? A. PSAs B. PSAEs C. PSRSs D. PSREs 5. These are issued to provide interpretive guidance and practical assistance to professional accountants in implementing PSAs and to promote good practice. A. PAPSs B. Practical Statements C. PRSPSs D. PAEPSs 6. Professional accountants should be aware of and consider Practice Statements applicable to the engagement. A professional accountant who does not consider and apply the guidance included in a relevant Practice Statement need not explain how the basic principles and essential procedures in the Engagement Standard(s) addressed by the Practice Statement have been complied with. A. Both statements are true. B. Both statements are false. C. True; False D. False; True 7. The Philippine Framework for Assurance Engagements defines and describes the elements and objectives of an assurance engagement, and identifies engagements to which PSAs, PSREs and PSAEs apply. The Philippine Framework for Assurance Engagements is not itself establish standards or provide procedural requirements for the performance of assurance engagements. A. Both statements are true. B. Both statements are false. C. True; False D. False; True 8. Which of the following statements best describes assurance services? A. Independent professional services that are intended to enhance the credibility of information to meet the needs of an intended user. B. Services designed to express an opinion on the fairness of historical financial statements based on the results of an audit. C. The preparation of the financial statements or the collection, classification, and summarization of other financial information. D. Services designed for the improvement of operations, resulting in better outcomes. 9. The following statements relate to the two types of assurance engagements that a practitioner is permitted to perform: reasonable assurance engagement and limited assurance engagement. Statement 1 The objective of a reasonable assurance engagement is a reduction in assurance engagement risk to an acceptably low level in the circumstances of the engagement as a basis for a positive form of expression of the practitioner’s conclusion. Statement 2 The objective of a limited assurance engagement is a reduction in assurance engagement risk to a level that is acceptable in the circumstances of the engagement, but where the risk is greater than for a reasonable assurance engagement, as the basis for a positive of expression of the practitioner’s conclusion. Statement 3 The objective of a reasonable assurance engagement is a reduction in assurance engagement risk to an acceptably low level in the circumstances of the engagement as the basis for a negative form of expression of the practitioner’s conclusion. Statement 1 Statement 2 Statement 3 A. False True True B. True False False C. True True False D. False False True 10. A practitioner who is engaged to perform an assurance engagement other than an audit or a review of historical financial information should comply with which of the following standards? A. PSAEs B. PSAEs and PSAs C. PSAs and PSREs D. PSAEs, PSREs and PSAs 11. Assurance engagements encompass the following types of services, except A. Attestation services B. Review engagements C. Audit of historical financial statements D. Management consulting 12. Assurance services differ from consulting services in that they I Focus on providing advice. II Involve monitoring of one party by another. A I only B. II only C. Both I and II D. Neither I nor II 13. Assurance engagements involve A. Two separate parties: a professional accountant and an intended user. B. Two separate parties: a professional accountant and a responsible party. C. Two separate parties: a responsible party and an intended user. D. Three separate parties: a professional accountant, a responsible party and an intended user. 14. An assurance engagement should have which of the following elements? I A three party relationship involving a practitioner, a responsible party and intended users. II An appropriate subject matter. III Suitable criteria. IV Sufficient appropriate evidence. V A written assurance report. A. I, II, and III only B. II, III, IV and V only C. I, II, III and IV only D. I, II, III, IV and V. 15. The subject matter of an assurance engagement may include A. Historical or prospective financial information. B. An entity’s internal control or IT system. C. Compliance with regulations. D. All of the above. 16. These are the benchmarks used to evaluate or measure the subject matter of an assurance engagement. A. Criteria B. GAAP C. Assertions D. Conclusions 17. Suitable criteria should have which of the following characteristics? I Relevance IV Neutrality II Completeness V Understandability III Reliability A. I, II, and III only B. II, III, IV and V only C. I, II, III and IV only D. I, II, III, IV and V. 18. In an assurance engagement, the person or persons, either as individuals or representatives of an entity, responsible for the subject matter is the A. Intended user B. Responsible party C. Practitioner D. Client 19. In an assurance engagement, the person or class of persons for whom the professional accountant prepares the report for a specific use or purpose is the A. Intended user B. Responsible party C. Management D. Client 20. An assurance engagement risk is the risk that the practitioner expresses an inappropriate conclusion when the subject matter information is materially misstated. An assurance engagement risk is the risk that the practitioner expresses an inappropriate conclusion when the subject matter information is not materially misstated. A. True; False B. Both statements are true C. False; True D. Both statements are false. 21. What level of assurance is provided by the auditor in an audit engagement? A. Absolute B High, but not absolute C. Moderate D. No assurance 22. What level of assurance is provided by the auditor in a review engagement? A. No assurance B. High, but not absolute C. Reasonable D. Moderate 23. What assurance is provided by the auditor in an agreed-upon procedures engagement? A. Reasonable B. Absolute C. Moderate D. No assurance 24. In a compilation engagement, the accountant is engaged to use accounting expertise as opposed to auditing expertise to collect, classify, and summarize financial information. What type of assurance is provided by the accountant when he/she performs this engagement? A. Positive assurance B. Negative assurance C. No assurance D. Limited assurance 25. The following statements relate to a review of financial statements. Which is incorrect? A. The objective of a review of financial statements is to enable an auditor to state whether, on the basis of procedures which do not provide all the evidence that would be required in an audit, anything has come to the auditor’s attention that causes the auditor to believe that the financial statements are not prepared, in all material respects, in accordance with an identified financial reporting framework. B. A review comprises inquiry and analytical procedures which are designed to review the reliability of an assertion that it is the responsibility of one party for use by another party. C. A review ordinarily involves an assessment of accounting and internal control systems, tests of records, and of responses to inquiries by obtaining corroborating evidence through inspection, observation, confirmation, and computation. D. The level of assurance provided in a review report is less than that given in an audit report. 26. In an engagement to perform agreed-upon procedures, an auditor is engaged to A. Carry out those procedures of an audit nature to which the auditor and the entity and any appropriate third parties have agreed and to report on factual findings. B. Use accounting expertise as opposed to auditing expertise to collect, classify, and summarize financial information. C. Provide a moderate level of assurance that the information is free of material misstatement. D. Provide a high, but not absolute, level of assurance that the information is free of material misstatement. 27. Which of the following statements concerning compilation engagement is incorrect? A. In a compilation engagement, the accountant is engaged to use accounting expertise as opposed to auditing expertise to collect, classify, and summarize financial information. B. The procedures employed in a compilation engagement enable the accountant to express a moderate level of assurance on the compiled financial information. C. Users of the compiled financial information derive some benefit as a result of the accountant’s involvement because the service has been performed with due professional skill and care. D. A compilation engagement ordinarily entails reducing detailed data to a manageable and understandable form without a requirement to test the assertions underlying that information. 28. The auditor should conduct an audit in accordance with Philippine Standards on Auditing. The auditor should plan and perform the audit with an attitude of professional skepticism. A. Both statements are true B. True; False C. Both statements are false. D. False; True 29. The primary reason for a financial statement audit by an independent CPA is to A. Provide increased assurance to users as to the fairness of the financial statements. B. Guarantee that there are no misstatements in the financial statements and ensure that any fraud will be discovered. C. Satisfy governmental regulatory requirements. D. Relieve management of responsibility for the financial statements. 30. Independent auditing can best be described as A. A branch of accounting. B. A professional activity that measures and communicates financial and business data. C. A discipline which attest to the results of accounting and other functional operations and data. D. A regulatory function that prevents the issuance of improper financial information. 31. Which of the following statements is correct concerning an auditor’s responsibilities regarding financial statements? A. An auditor’s responsibilities for audited financial statements are confined to the expression of the auditor’s opinion. B. The fair presentation of audited financial statements in conformity with GAAP is an implicit part of the auditor’s responsibilities. C. Making suggestions that are adopted about the form and content of an entity’s financial statements impairs an auditor’s independence. D. The auditor’s report should provide an assurance as to the future viability of the entity. Internal auditing involves appraising the economy and efficiency with which resources are employed. B. 37. Operational audits generally have been conducted by internal and COA auditors. To serve as the investigative arm of he of the audit committee of the board of directors. and also A. Which of the following is considered a primary reason for creating an internal audit department? A. A governmental audit may extend beyond an audit leading to the expression of opinion on the fairness of financial presentation to include Program Results Compliance Economy and Efficiency A. C. C. B. To relieve management of the responsibility for establishing effective controls. 33. reliability. Which of the following best describes the scope of internal auditing as it has developed to date? A. C. Evaluation D. A measure of management performance in meeting organizational goals. An objective of a performance audit is to determine whether an entity’s A. D. Yes Yes Yes B. C. D. In conducting an appraisal of the economy and efficiency with which company resources are used. Operational information is in accordance with government auditing standards. To safeguard resources entrusted to the organization. objective assurance and consulting activity that adds value to operations. and timeliness of financial and operating data used in management’s decision making. B.32. Accuracy B. Internal auditing has evolved to verifying the existence of assets and reviewing the means of safeguarding assets. an internal auditor’s responsibility is to A. Internal auditing has evolved to more of an operational orientation from a strictly financial orientation. Aid to the independent auditor. To evaluate and improve the effectiveness of control processes. 35. Internal Control 38. effectiveness. To assist the external auditor to reduce external audit fees. A means of assurance that internal accounting controls are functioning as planned. Determine whether operating standards have been established. What is the proper organizational role of internal auditing? A. Compliance C. D. To ensure the accuracy. . but may be performed by certified public accountants. To perform studies to assist in the attainment of more efficient operations. Verify the existence of assets. Yes Yes No C. B. D. economy. Review the reliability of operating information. Verify the accuracy of asset valuation. Internal auditing has evolved to verifying the existence of assets and reviewing the means of safeguarding assets. D. A primary purpose of an operational audit is to provide A. No Yes Yes 39. To serve as an independent. Yes No Yes D. The results of internal examinations of financial and accounting matters to a company’s top-level management. Governmental auditing often extends beyond examinations leading to the expression of opinion on the fairness of financial presentation and includes audits of efficiency. C. B. who is conducting the examination of the financial statements. 36. 34. A practitioner should obtain sufficient relevant data to complete the engagement. including those who provide audit and tax services. 40. B. Which of the following are considered consulting services? Advisory Services Transaction Services Assurance Services A. Which of the following statements concerning consulting services is false? A. Cost accounting procedures. A practitioner is to maintain an appearance of independence. Financial statements present fairly the results of operations. B. Most CPAs.B. She has little experience with the industry involved. 45. Consulting services ordinarily involve external reporting. Decline the engagement because he lacks sufficient knowledge. C. Independent practitioner. Accept the engagement and perform it in accordance with PSAs. D. C. D. 42. Which of the following statements applies to consultation services engagements? A. and Other Assurance and Related Services  PSA 220 (revised) Quality Control for Audits of Historical Financial  PSA 210 (amended by PSA 700 revised) Terms of Audit Engagements . A pervasive characteristics of a CPA’s role in a consulting services engagement is that of being a (an) A. Computer expert. ---------------------------------------------------------------------------------------------------------  PSQC1 Quality Control for Firms that Perform Audits and Reviews of Historical Financial Information. B. The performance of consulting services for audit clients does not. D. an internal auditor is more likely to be concerned with A. In comparison to the independent auditor. What is his most appropriate action? A. A practitioner is not permitted to compile a financial forecast. Legal and regulatory compliance. Yes Yes No C. D. A practitioner should obtain an understanding of the internal control to assess control risk. C. C. Accept the engagement and issue a report vouching for the achievability of the results of the merger. B. B. Confidential reviewer. CPA. Yes Yes Yes 44. Reyes. in and of itself. C. Objective advisor. impair the auditor’s independence. Operational auditing. Specific operating units are functioning economically and efficiently. 43. Accept the engagement and perform additional research or consult with others to obtain sufficient competence. No Yes Yes B. has been asked to perform a consulting services engagement concerning the analysis of a potential merger. Internal control is adequately operating as designed. 41. Internal control system. Consulting services differ fundamentally from the CPA’s function of attesting to the assertions of other parties. also provide consulting services to their clients. D. C. D. Yes No Yes D. D. The primary purpose of establishing quality control policies and procedures for deciding whether to accept a new client is to A. Continuing professional education. Maintaining personnel files containing documentation related to the evaluation of personnel. Satisfy the CPA firm’s duty to the public concerning the acceptance of new clients. This statement defines the quality control element of A. Evaluating prospective and continuing client relationships. Engagements selected for inspection include . the firm most likely would establish procedures for A. According to PSQC 1. A CPA firm’s quality control procedures pertaining to the acceptance of a prospective audit client would most likely include A. Compliance with generally accepted reporting standards. B. including a periodic inspection of a selection of completed engagements is ordinarily performed on a cyclical basis. about information regarding the prospective client and its management. Minimize the likelihood of association with clients whose management lacks integrity. B. B. C. such as the prospective client’s bankers and attorneys. Compliance with the fundamental principles of the Philippine Code of Ethics. Consideration of whether internal control is sufficiently effective to permit a reduction in the extent of required substantive tests. 7. One of a CPA firm’s basic objectives is to provide professional services that comply with professional standards and regulatory and legal requirements. Monitoring. B. a firm should establish policies and procedures to provide it with reasonable assurance that the policies and procedures relating to the system of quality control are relevant. D. To achieve this goal. adequate. Reviewing engagement working papers and reports. Acceptance and continuance of client relationships and specific engagements. 2. Requiring personnel to adhere to the applicable independence rules. D. Consideration of whether sufficient appropriate evidential matter may be obtained to afford a reasonable basis for an opinion. Inquiry of management as to whether disagreements between the predecessor auditor and the prospective client were resolved satisfactorily. Reasonable assurance of achieving this basic objective is provided through A. A system of quality control. A CPA firm should establish procedures for conducting and supervising work at all organizational levels to provide reasonable assurance that the work performed meets the firm’s standards of quality. No Yes Yes D. C. Inquiry of third parties. Which of the following are elements of a CPA firm’s quality control that should be considered in establishing its quality control policies and procedures? Human Resources Monitoring Engagement Performance A. 4. Yes No Yes 3. Yes Yes No B. D. Enable the CPA firm to attest to the reliability of the client. Leadership responsibility for quality within the firm. Yes Yes Yes C. C. Anticipate before performing any field work whether an unqualified opinion can be expressed.1. Such policies and procedures should include an ongoing consideration and evaluation of the firm’s system of quality control. Ethical requirements. C. 6. operating effectively and complied with in practice. 5. D. C. Quality control policies and procedures should provide the firm with reasonable assurance that the policies and procedures relating to the other elements of quality control are being effectively applied. B. Weekly C. C. and V only. I. II.A. the engagement partner should A. of the significant judgments the engagement team made and the conclusions they reached in formulating the report. The significant judgments made by the engagement team. D. including those identified during the engagement quality control review. B. A firm’s system of quality control should ordinarily provide for the maintenance of A. An engagement quality control review should include an objective evaluation of I. D. B. Monitoring. 8. IV. 11. II The conclusions reached in formulating the auditor’s report. and V only. PSA 220 (revised) requires the engagement partner to consider whether members of the engagement team have complied with the ethical requirements relating to audit engagements of the Philippine Code. Engagement performance. before the report is issued. the firm should communicate the results of the monitoring of its quality control system to engagement partners and other appropriate individuals within the firm at least A. and V only. D. C. . At least one engagement for each engagement partner over an inspection cycle. A file of minutes of staff meetings. D. C. I only B. C. The Philippine Code establishes the fundamental principles of professional ethics. As defined in PSQC 1. At least one engagement for each engagement partner over an inspection cycle. 13. Under PSQC 1. B. Monthly B. Annually 9. IV. For audits of financial statements of listed entities. III. B. which ordinarily spans no more than 2 years. All of the above. IV. I. At least 3 engagement for each engagement partner over an inspection cycle. which ordinarily spans no more than 5 years D. II. Both I and II. 10. Neither I nor II. II. A. Discuss significant matters arising during the audit engagement. Updated personnel files. Continuing professional education. which ordinarily spans no more than 3 years. Determine that an engagement quality control reviewer has been appointed. B. Quarterly D. 12. it is a process designed to provide an objective evaluation. IV and V. I. which include I Integrity II Objectivity III Professional competence and due care IV Confidentiality V Professional behavior A. III. A. Not issue the auditor’s report until the completion of the engagement quality control review. At least one engagement for each engagement partner over an inspection cycle. III. II only C. which ordinarily spans no more than 1 year. with the engagement quality reviewer. Engagement quality control review. The audit work performed by each assistant should be reviewed to determine whether it was adequately performed and to evaluate whether the A. Our engagement is subject to the risk that material errors or fraud.Benefit Firm’s Size CPA Firm’s Practice Considerations A. D. Yes Yes Yes B. Management is responsible for making all financial records and related information available to us. Which of the following statements would least likely appear in an auditor’s engagement letter? A. A. D.” The above passage is most likely from a/an A. branch or division (component) should send a separate engagement letter to the entity’s component. Engagement letter. C. if they exist. The engagement letter documents and confirms the A B C D Auditor’s acceptance of the appointment Yes Yes Yes Yes Objective and scope of the audit Yes No Yes Yes Extent of the auditor’s responsibilities to the client No Yes No Yes Form of any reports Yes No No Yes 17. Audit procedures performed are approved in the professional standards. Audit has been performed by persons having adequate technical training and proficiency as auditors. B. The auditor of a parent entity who is also the auditor of its subsidiary. B. Yes No Yes D. Documentation to provide evidence of the operation of each element of its system of quality control. the auditor should send a new engagement letter each period. Results are consistent with the conclusions to be presented in the auditor’s report. Both statements are true. An auditor’s document includes the following: “We will conduct our audit in accordance with Philippine Standards on Auditing. 19. we will discuss with you the other procedures we consider necessary to complete the engagement. C. will not be detected. Letter of acceptance C. No Yes Yes 16. Those Standards require that we comply with ethical requirements and plan and perform the audit to obtain reasonable assurance whether the financial statements are free from material misstatement. B. On recurring audits. D. Both statements are false. False . True. 14. C. D. B. 18. Emphasis-of-matter paragraph of a “subject to” qualified auditor’s report. plus travel and other out-of-pocket expenses. 15 The nature and extent of a CPA firm’s quality control policies and procedures depend on The CPA The Nature of the Cost. Documentation to demonstrate compliance with regulatory requirements. Auditor’s system of quality control has been maintained at a high level. Fees for our services are based on our regular per diem rates. After performing our preliminary analytical procedures. Management representation letter. Yes Yes No C.C. incomplete or otherwise unsatisfactorily. An auditor who. Both statements are true. timing and direction of the audit guides the development of the more detailed overall audit strategy. A. Reporting B. C. . Organizing 2. B. is requested to change the engagement to one which provides a lower level of assurance. Adequate planning of the audit work helps to ensure that A B C D Appropriate attention is devoted to important areas of the audit Yes No Yes No Potential problems are identified Yes Yes Yes No The work is completed expeditiously No Yes Yes No 3. should A. C. False: True 20. Field work D. True. The extent of planning will vary according to the A B C D Size of the entity Yes Yes No No Complexity of the audit Yes No Yes No Auditor’s experience with the entity and knowledge of the business Yes Yes Yes No 4.  PSA 300 (rev. False D. Obtaining knowledge of the entity’s business is an important part of planning the audit work. The auditor’s knowledge of the entity’s business assists in the identification of events. Planning C. According to PSA 300. The audit plan sets the scope. Withdraw and consider whether there is any obligation to report to other parties the circumstances necessitating the withdrawal.D. The auditor should plan the audit so that the audit will be performed in an effective manner. transactions and practices which may have a material effect on the financial statements A. the auditor may discuss elements of planning with those charged with governance and the entity’s management. Both statements are false. before the completion of the engagement. According to PSA 300. Issue a report that includes reference to the original engagement and any procedures that may have been perform in the original engagement. B. Not agree to a change of engagement where there is no reasonable justification for doing so. Consider the change reasonable if it relates to information that is incorrect. D. False: True 5.) Planning an Audit of Financial Statements  PSA 315 UNDERSTANDING THE ENTITY AND ITS ENVIRONMENT ASSESSING THE RISKS OF MATERIAL MISSTATEMENT 1. it involves establishing the overall audit strategy for the engagement and developing an audit plan in order to reduce audit risk to an acceptably low level. A.The overall audit strategy is more detailed than the audit plan and includes the nature. Control activities . control risk. timing and extent of audit procedures to be performed by engagement team members to obtain sufficient appropriate audit evidence to reduce audit risk to an acceptably low level. Detection risk C. True. such as the existence of related parties. are the results of complex calculations. B. Cash is more susceptible to theft than an inventory of coal because it has a greater inherent risk. D. control risk. It sets the tone of the organization. its financial performance and its reporting requirements including changes since the date of the prior audit. C. and detection risk can be changed at the discretion of the auditor. A. False C. 7. True. A. It is the process designed and effected by those charged with governance. The setting of materiality levels for audit purposes. and detection risk. Yes Yes Yes C. Risk of failing to discover material misstatements. False. The susceptibility to material misstatements in these types of accounts is referred to as A. False. D. No No No D. its business. No No Yes 9. provides discipline and structure. Control risk 10. C. Audit risk has three components: inherent risk. The risk that material misstatements will not be prevented or detected on a timely basis by internal control can be reduced to zero by effective controls. Which of the following matters should be considered by the auditor in developing the overall audit strategy? A. True B. Important characteristics of the entity. False. 11. Which of the following statements is correct? A. Inherent risk D. Conditions requiring special attention. True. Yes No Yes B. Audit risk B. B. There is an inverse relationship that exists between the acceptable level of detection risk and the A. All of the above. management. 8. C. False 6. and other personnel to provide reasonable assurance about the achievement of the entity’s objectives. Preliminary judgments about materiality levels. Internal control C. False. Internal auditing B. True. Business strategy D. and influences the control consciousness of employees. Detection risk is a function of the efficiency of an auditing procedure. This internal control component is the foundation for all other components. False D. Accounting process 12. Risk of misapplying audit procedures. Some account balances. such as those for retirement benefits and finance leases. Assurance provided by substantive tests. B. Which of the following audit risk components may be assessed in quantitative terms? Inherent Risk Control Risk Detection Risk A. The existing levels of inherent risk. D. No No Yes 14. while controls over the reliability of financial reporting may also be relevant.B. it is important for the auditor to consider the competence of its employees. The accounting manager reviews al accounting transactions. Cost-benefit relationship of internal control. Comparison of recorded accountability with assets. The audit of the annual financial statements. Which of the following are considered control environment elements? Commitment to Competence Detection Risk Organizational Structure A. Is unaffected by changing circumstances and conditions encountered by the entity. Monitoring of controls is accomplished through ongoing monitoring activities. Control environment D. It involves assessing the design and operations of control on a timely basis and taking necessary corrective actions. Controls over safeguarding of assets and liabilities are of primary importance. because their competence bears directly and importantly upon the A. C. When obtaining knowledge about an entity’s internal control. Segregation of duties B. monitoring of controls is an internal control component that involves a process of assessing the quality of internal control performance of time. B. Control activities constitute one of the five components of internal control. Eliminates risk and potential loss to the entity. separate evaluations. Which of the following is not included in this internal control component? A. Controls over the reliability of financial reporting are ordinarily most directly relevant to a financial statement audit. Periodic reporting by the entity’s internal auditors about the functioning of internal control. 15. C. Reviewing the purchasing function. Corporate morale problems are addressed immediately and effectively. All controls are ordinarily relevant to a financial statement audit. An internal audit function D. 13. D. 18. D. D. Under PSA 315. 16. Monitoring of controls C. Financial statements are fairly presented. but other controls may also be relevant. Yes No Yes D. Reduces the need for management to review exception reports on a day-to-day basis. . An internal control system that is working effectively A. Which of the following statements concerning the relevance of various types of controls to a financial statements audit is correct? A. Authorization 17. B. C. No Yes No B. D. one of the auditor’s major concerns is to ascertain whether internal control is designed to provide reasonable assurance that A. When considering an entity’s system of internal control. Cannot be circumvented by management. Profit margins are maximized. B. Periodic audits by the audit committee. An entity’s ongoing monitoring activities often include A. or a combination of the two. B. Performance reviews C. 19. Yes Yes Yes C. and operational efficiency is optimized. B. The entity’s risk assessment process. C. An auditor may ordinarily ignore a consideration of controls when a substantive audit approach is taken. and disclosures. D. Controls related to each class of transactions and account balance. Evidence to use in reducing detection risk. Consistency with which controls are currently being applied. I. Observing employees as they apply specific controls. 23. an auditor is required to obtain knowledge about the A. Knowledge necessary to plan the audit. timing. Effectiveness of controls that have been implemented. require special audit consideration. Authorizes a transaction and maintains custody of the asset that resulted from the transaction. in the auditor’s judgment. Procedures used to assure proper authorization of transactions. including the related business processes relevant to financial reporting. Such risks are called A. I and III only. Records a transaction does not compare the accounting record of the asset with the asset itself. An auditor should obtain sufficient knowledge of an entity’s information system. II. B. C. B. C. B. 26. such as those that involve fraud or complex transactions. to understand the A. and extent of further audit procedures. D. D. Process used to prepare significant accounting estimates. Control activities are the policies and procedures that help ensure that management directives are carried out. C. the auditor is required to identify and assess the risks of material misstatements at the financial statement level. A. There is proper segregation of duties when an individual who A. C. and segregation of duties. 20. Safeguards used to limit access to computer facilities. In obtaining an understanding of internal control relevant to the audit. D. Design of the controls pertaining to internal control components. and at the assertion level for classes of transactions. Achievement of the objectives of internal control. account balances. II Consider factors that affect the risks of material misstatements. information processing. These include activities relating to authorization. B. D. Information necessary to prepare flowcharts. C. The auditor uses the understanding of internal control to I Identify types of potential misstatements. A basis for modifying tests of controls. C. Obtaining a flowchart of activities performed by entity personnel. Maintains custody of an asset and has access to the accounting records for the asset. performance reviews. Timing of substantive tests to be performed. Authorizes a transaction and records it. Information about segregation of duties ordinarily is best obtained by A. B. Policies used to detect the concealment of fraud. Business risks . 24. 22. D. I and II only. Performing tests of transactions that corroborate management’s financial statements assertions. The primary objective of procedures performed to obtain an understanding of internal control is to provide an auditor with A. and III 25. Some of these risks. Developing audit objectives that reduce control risk. 21. B.C. physical controls. In conducting an audit in accordance with PSAs. II and III only D. III Design the nature. Shipping department supervisor 30. Understatement Understatement Understatement B. As a result of obtaining an understanding of an entity’s internal control system. Receiving clerk B. D. Each shipment of goods on credit is supported by a prenumbered sales invoice. Those charged with governance or management B. The billing department supervisor sends copies of approved sales orders to the credit department for comparison to authorized credit limits and current customer account balances. Ascertain that raw materials paid for are on hand. C. C. Overstatement Overstatement Understatement C. Material risks 27. Determine the reliability of financial reporting by the purchasing function. The accounting department supervisor independently reconciles. The billing department supervisor matches prenumbered shipping documents with entries in the sales journal. 29. 33. 32. C. except A. A sound internal control procedure should require that defective merchandise returned by customers be presented initially to the A. the accounts receivable subsidiary ledger to the accounts receivable control account. The inadequate controls could cause what type of misstatement in each of the following accounts? Revenues Receivables Inventories A. Which of the following controls most likely would provide reasonable assurance that all credit sales transactions of an entity are recorded? A. B.B. Macho Dancer Company uses its sales invoices for posting perpetual inventory records. B. . The accounts receivable subsidiary ledger is reconciled daily to the accounts receivable control account in the general ledger. Each sales invoice is supported by a prenumbered shipping document. Effective controls relevant to purchasing of raw materials should usually include all of the following. the auditor may become aware of material weaknesses in the design or implementation of internal control. The accounting department supervisor controls the mailing of monthly statements to customers and investigates any differences reported by customers. Overstatement Overstatement Overstatement 31. Audit risks C. D. Inadequate internal control over the invoicing function allows goods to be shipped but not invoiced. B. Billing clerk D. The auditor is required to communicate this matter to A. Chief executive officer C. Observe the annual physical count. Board of Accountancy 28. Daily sales summaries are compared to daily postings to the accounts receivable ledger. Accounts receivable supervisor C. Investigate the recording of unusual transactions regarding raw materials. Understatement Understatement Overstatement D. Which of the following control activities in an entity’s revenue/receipt cycle would provide reasonable assurance that all billed sales are correctly posted to the accounts receivable ledger? A. on a monthly basis. The auditor’s primary objective in obtaining an understanding of the client’s controls over the purchasing function is to A. Securities and Exchange Commission D. D. Significant risks D. Determining the need for the raw materials prior to preparing the purchase order. Establishing a cutoff for goods received and shipped. Cash receipts are not deposited intact daily. Treasurer does not verify the names and addresses of check payees. Obtaining financial approval prior to making a commitment. regardless of cost-benefit considerations.5% of purchases by designing and implementing certain controls costing approximately P350. C. D. Yes. Inventories are the most liquid assets. 38. 39. Are prenumbered purchase orders and receiving reports used and accounted for? D. C. because the ideal system of internal control is the most extensive one. Indicating on the voucher the affected asset and expense accounts to be debited. has annual sales of P30. 37. . Obtaining third-party written quality and quantity reports prior to payment for the raw materials. and receiving report. 35. Which of the following is an essential control procedure to ensure the accuracy of the recorded inventory quantities? A. Your client. Which of the following controls most likely would be implemented to achieve the production cycle control objective of maintaining accurate inventory records? A. B. Are intact cash receipts deposited daily in the bank? C. B. Are an approved purchase requisition and a signed purchase order required for each purchase? 36. and compared with purchase orders on receipt? B. D. Checks are signed by one person. Tests reveals that 2% of the peso amount of purchases do not get into inventory because of breakage and inventory pilferage by employees. Signed checks are distributed by the controller to approved payees. Matching the vendor’s invoice with the related purchase requisition. 34. The following are appropriate questions on an internal control questionnaire concerning purchase transactions. B. Systematic reporting of product changes that will affect raw materials. A just-in-time inventory ordering system keeps inventory levels to a desired minimum. Inventories typically represent a large component of an entity’s current assets. D. C.000. purchase order.B.000 and a 40% gross profit rate. D. The company estimates that these losses could be reduced to 0. Inventories directly affect the financial performance of an entity. C. No. Periodic inventory counts are used to adjust the perpetual inventory records. Approving vouchers for payment by having an authorized employee sign the vouchers. except A. Accounting for unused prenumbered purchase orders and receiving reports. Which of the following controls is not usually performed in the accounts payable department? A. Effective internal controls over inventories are designed and implemented for the following reasons. Inventories typically represent a large portion of an entity’s total assets. Yes. C. Calculating unit costs and valuing obsolete or damaged inventory items in accordance with inventory policy. Which of the following is of least concern to an auditor in assessing the risks of material misstatement? A. Testing inventory extensions. B. because the cost of designing and implementing the added controls exceeds the projected savings. Yes. because the situation involves employee theft. Performing a gross profit test. a merchandising concern. 40. because the expected benefits to be derived exceed the cost of the added controls. Should the controls be designed and implemented? A. D. inspected. C. except A. Are all goods received in a centralized receiving department and counted. D. B.000. B. Recently acquired loans include covenants that preclude further plant acquisitions for 5 years.C. Purchase requisitions. C. Securities should be registered in the entity’s name. The following controls are designed to protect investment securities. Analyze the liquidity and turnover ratio of the inventory. An internal control objective concerning property. plant. Custody over investment securities should be limited to personnel having record-keeping responsibility over the securities. Preparation of payroll transaction journal entries by an employee who reports to the supervisor of the personnel department. B. D. Verification of agreement of job time tickets with employee clock card hours by a payroll department employee. 45. Effective controls over the payroll function may include A. Which of the following controls would an entity most likely used in safeguarding against the loss of trading securities? A. B. Reconciliation of totals on job time tickets with job reports by employees responsible for those specific job. D. An independent trust company that has no direct contact with the employees who have record keeping responsibilities has possession of the securities. D. 44. C. All material acquisitions of PPE are required to be approved by the board of directors. plant. D. C. Gross PPE increased 30% during the current period. D. Perpetual inventory records are periodically compared with the net realizable value of individual inventory items. C. B. The independent auditor traces all purchases and sales of trading securities through the subsidiary ledgers to the general ledger. Proper authority for acquisition and retirement of PPE items. purchase orders. 42. B. and equipment (PPE) acquisitions is that they be recorded at the correct amounts and in the proper period. . Review the entity’s description of inventory policies and procedures. A designated member of the board of directors controls the securities in a bank safe- deposit box. In which of the following conditions would an auditor most likely to assess a high level of risk of material misstatements? A. A. Perform test counts of inventory when observing the entity’s physical count. except. C. C. Custody of rate authorization records by the supervisor of the payroll department. D. B. B. Detailed PPE records and physical controls over PPE items. Perform analytical procedures designed to identify significant cost variances. Most additions are self-constructed by the entity. and equipment (PPE). The following controls are appropriate for property. Which of the following is the most likely procedure an auditor would perform in obtaining an understanding of a manufacturing entity’s internal control for inventory balances? A. 43. The internal auditor inspects the trading securities in the entity’s safe each year on the balance sheet date. Access to securities should be vested in two individuals. Disposal of fully depreciated PPE items. receiving reports. D. and vendor invoices are independently matched before payment is approved. and properly classified. 46. Investment securities should be properly controlled physically in order to prevent unauthorized usage. 41. Written policies for capitalization and expenditure and review of application of depreciation methods. except A. Which of the following personnel department procedures reduces the risk of payroll fraud and represents an appropriate responsibility for the department? A. B. Payroll records are periodically reconciled with tax reports. Authorizing overtime hours. Which of the following situations represents an internal control weakness in the payroll department? A. Control risk should be assessed in terms of A. Either I or II D. B. C. Distributing paychecks. Which of the following statements concerning the use of these documents is incorrect? A. When substantive procedures alone do not provide sufficient appropriate Audit evidence at the assertion level. C. Tests of controls are concerned primarily with each of the following questions. Payroll department personnel are rotated in their duties. D. Types of potential fraud. Each employee should have only one time card. Payroll should be calculated based on job time tickets. Employees of a manufacturing entity are often required to use time cards and job time tickets. By whom were the controls applied? B. D. Which one of the following functional separations is not required for internal control purposes? A. In a financial statement audit. Authorizing the addition or deletion of employees from the payroll. Why were the controls? 3. except A. Were the necessary controls consistently performed? C. Neither I nor II 2. C. I only B. record-keeping. D. II. Separation of personnel function from payroll preparation. Specific control procedures. Control environment factors. Collection and retention of unclaimed paychecks. 50. D. Organizational independence in the processing of payroll can be achieved by segregating the functions of authorization. Time reported on job time tickets should be reconciled to time cards. A. How were the controls applied? D. Paychecks are distributed by the employees’ immediate supervisor. -------------------------------------------------------------------------------------------------------------------  PSA 330 THE AUDITOR’S PROCEDURES IN RESPONSE TO ASSESSED RISKS 1. 49. B. Separation of timekeeping from payroll preparation. C. Financial statement assertions. D. 48. An employee may have one or many job time tickets in a day. C. Separation of payroll preparation and paycheck distribution. II only C. .47. the auditor is required to perform tests of controls when I. B. B. and custody of assets. Separation of payroll preparation and maintenance of year-to-date records. The timekeeping function is independent of the payroll department. The auditor’s risk assessment includes an expectation of the operating effectiveness of controls. Controls are unlikely to pertain to the assertions. Which of the following statements is correct concerning an auditor’s assessment of control risk? A. The entity’s control components are interrelated. an auditor who plans to rely on controls that have not changed since they were last tested should test the operating effectiveness of such controls at least once every A. Analytical procedures and confirmation. 5. 8. C. Confirmation and observation. The following statements relate to the use of audit evidence when testing the operating effectiveness of relevant controls. The lower the assessed level of control risk. More emphasis on tests of controls than substantive tests is warranted. B. Fourth audit D. D. Audit evidence pertaining only to appoint in time may be sufficient for the auditor’s purpose. Third audit C. C. Which is false? A. D. Fifth audit . D. Additional evidence to support a further reduction in control risks is not available. If an auditor plans to rely on controls that have changed since they were last tested. the less assurance the evidence must provide that the controls are operating effectively. Evidence about the operation of controls in prior audits may not be considered during the current year’s assessment of control risk. the auditor should test the operating effectiveness of such controls in the current audit. 6. Assessing control risk may be performed concurrently during an audit with obtaining an understanding of the entity’s internal control. when testing controls over an entity’s physical count of inventories at year-end. B. D. C. It is not possible or practicable to reduce the risks of material misstatement at the assertion level to an acceptably low level with audit evidence obtained only from substantive test procedures.4. for example. There were many internal control weaknesses that could allow misstatements to enter the accounting system. If the control procedures leave no audit trail of documentary evidence. Inquiry and analytical procedures. After gaining an understanding of internal control and assessing the risks of material misstatement. Sufficient appropriate audit evidence to support the assertions is likely to be available. B. An auditor may plan to use audit evidence about the operating effectiveness of controls obtained in prior audits. An auditor who obtains sufficient appropriate audit evidence about the operating effectiveness of controls during the interim period should no longer obtain additional evidence of operating effectiveness for the remaining period. 9. 7. an auditor decided to perform tests of controls. Inquiry and observation. An increased in the assessed level of control risk is justified for certain financial statement assertions. An auditor may decide to assess control risk at the maximum level for certain assertions because the auditor believes A. According to PSA 330 (The Auditor’s Procedures in Response to Assessed Risks). D. C. Second audit B. the auditor most likely will test the procedures by A. B. B. An auditor intends to perform tests of control on a client’s cash disbursements procedures. C. The basis for an auditor’s conclusions about the assessed level of control risk need not be documented unless control risk is assessed at the maximum level. The auditor most likely decided that A. 16. Decrease detection risk. Decrease inherent risk. Analytical procedures and confirmation. Confirmation and observation. C. including A. Level of detection risk. Inspect unused sales invoices for consecutive prenumbering. Extent of tests of details. the auditor would A. Periodically submitting auditor-prepared test data to the same computer process and evaluating the results. an auditor selects from a variety of techniques. Regardless of the assessed level of control risk. Dual-purpose tests to evaluate both the risk of monetary misstatements and preliminary control risk. B. 12. an auditor cannot verify the reliable operation of programmed control procedures by A. B. the auditor would most likely increase the A. D. 14. Increase inherent risk. the processing of input data and comparing the simulated results with the actual results. Reperformance and observation. Extent of tests of controls. In performing tests of the operating effectiveness of an entity’s controls. B. Manually comparing detail transaction files used by an edit program with the program’s generated error listings to determine that errors were properly identified by the edit program. D. C. C. B. D. B. Constructing a processing system for accounting applications and processing actual data from throughout the period through both the client’s program and the auditor’s program. C. C. If the control procedures leave no audit trail of documentary evidence. Which of the following tests of controls most likely would help assure an auditor that goods shipped are properly billed? A. 15. Level of inherent risk. 13. Inquiry and analytical procedures. Analytical procedures to verify the design of internal control procedures. D. Inspection and verification. as of a moment in time. B. 11. . the auditor most likely will test the procedures by A. Inquiry and observation. On the basis of audit evidence gathered and evaluated. Increase materiality level. C. C. An auditor intends to perform tests of control on a client’s cash disbursement procedures. D. B. Manually reperforming. When an accounting application is processed by computer. Tests of controls to determine the effectiveness of internal control policies. To achieve an overall audit risk level that is substantially the same as the planned audit risk level. Inquiry and analytical procedures. an auditor would perform some A. an auditor decides to increase the assessed level of control risk from that originally planned. D. When an auditor increases the planned assessed level of control risk because certain controls were determined to be ineffective. Comparison and confirmation. Examine shipping documents for matching sales invoices. Scan the sales journal for sequential and unusual entries. D.10. Substantive tests to restrict detection risk for significant transaction classes. Compare the accounts receivable ledger to daily sales invoices. An auditor uses the knowledge provided by the understanding of internal control and the final assessed level of control risk primarily to determine the nature. Which of the following procedures concerning accounts receivable would an auditor most likely to perform to obtain evidential matter in support of an assessed level of controls risk below the maximum level? A.17. Comparing an entity’s uncollectible accounts expense to actual uncollectible accounts receivable. C. Attorney’s responses to the auditor’s inquiries. C. D. Classification of revenue and expense transactions by product line. C. Analytical procedures for current year property and equipment transactions. Select and examine cancelled checks and ascertain that the related receiving reports are dated no earlier than the checks. Select and examine receiving reports and ascertain that the related canceled checks are dated no later than the receiving reports. D. C. Tests of controls D. plant and equipment balances at the end of the year. When there are numerous property and equipment transactions during the year. Sending confirmation request’s to an entity’s principal customers to verify the existence of accounts receivable. Inspecting an entity’s analysis of accounts receivable for unusual balances. Which of the following procedures provides the greatest assurance that this control is operating effectively? A. An internal control questionnaire indicates that an approved receiving report is required to accompany every check request for payment of merchandise. 18. Tests of controls and limited tests of current year property and equipment transactions. Confirmations of receivables verifying account balances. B. and extent of the A. 23. B. C. Tests of controls are least likely to be omitted with regard to A. C. Which of the following types of evidence would an auditor most likely examine to determine whether controls are operating as designed? A. Subsequent events. Accounts representing many transactions. Compliance tests C. Tests of controls and extensive tests of property. . D. B. Accounts representing few transactions. timing. D. Client records documenting the use of computer programs. Letters of representations corroborating inventory pricing. Approval of the purchase and sale of trading securities. D. Analytical procedures for property and equipment balances at the end of the year. 22. Observing an entity’s employee prepare the schedule of past due accounts receivable. Comparison of receiving reports and vendor’s invoices with purchase orders. 21. Accounts believed to be subject to ineffective controls. Segregation of the functions of recording disbursements and reconciling the bank account. D. B. Substantive tests 19. an auditor who plans to assess control risk at a low level usually performs A. Attribute tests B. Select and examine cancelled checks and ascertain that the related receiving reports are dated no later than the checks. B. 20. Select and examine receiving reports and ascertain that the related canceled checks are dated no earlier than the receiving reports. B. An auditor is least likely to test controls that provide for A. the independent auditor should discuss with management the effectiveness of the company’s controls that protect against the purchase of A. In evaluating the fair presentation of the financial statements. Specific misstatements identified by the auditor including the net effect of uncorrected misstatements identified during the audit of previous period. timing and extent of audit procedures. Required supplies provided by vendor who offers no trade discounts. C. A. Based on observations made during an audit. False. Both I and II.. A. True. C. False. Required supplies provided by vendor who offers no cash discounts.24. True. without considering possible volume discounts. True 2. The existence within the purchasing cycle of internal control strengths that offset weaknesses. In assessing control risk for the purchasing cycle. B. Supplies individually ordered. e. A. the auditor should assess whether the aggregate of uncorrected misstatements that have been identified during the audit is material. The auditor’s best estimate of other misstatements which cannot be specifically identified. D. False. II only C. The results of the internal control questionnaire. The audit work performed in the purchasing cycle by the company’s internal auditor. True. True D. 4. B. The entity’s annualized interim financial statements. The anticipated sample size of the planned substantive tests. D. the auditor will be least influenced by A. Neither I nor II. There is an inverse relationship between materiality and the level of audit risk. D. The effectiveness of the controls in other cycles. B. False B. I only B. Inventory items acquired based on an economic order quantity (EOQ) inventory management concept. 3. 25. II. Materiality need not be considered when evaluating the effect of misstatements. C. True. The availability of a company manual describing policies and procedures for the purchasing cycle.g. True C. A basic premise underlying analytical procedures is that . The aggregate of uncorrected misstatements comprises I. The contents of the management representation letter.  PSA 320 AUDIT MATERIALITY  PSA 520 ANALYTICAL PROCEDURES  PSA 550 RELATED PARTIES  PSA 610 CONSIDERING THE WORK OF INTERNAL AUDIT  PSA 620 USING THE WORK OF AN EXPERT Materiality should be considered by the auditor when determining the nature. Which of the following would an auditor most likely use in determining the auditor’s preliminary judgment about materiality. D. the sales-receivables-cash receipts cycle. False. A. The primary objective of analytical procedures used in the overall review stage of an audit is to A. 5. Statistical tests of financial information may lead to the discovery of material misstatements in the financial statements. 8. C. D. Analytical procedures may be omitted entirely for some financial statement audits. C. No Yes Yes B. Transactions subject to management discretion. B. Projecting an error rate by comparing the results of a statistical sample with the actual population characteristics. B. C. Examining computer-generated exception reports to verify the effectiveness of internal control. C. D. Analytical procedures used in planning the audit should not use nonfinancial information. Which of the following items tend to be the most predictable for purposes of analytical procedures applied as substantive tests? A. Which of the following procedures would an auditor most likely to perform in planning a financial statement audit? A. Comparing the financial statements with anticipated results. D. Assist the auditor in assessing the validity of the conclusions reached. B. Yes Yes No C. Computing accounts receivable turnover by dividing credit sales by the average net receivables. . Estimating payroll expense by multiplying the number of employees by the average hourly wage rate and the total hours worked. D. 10. the use of analytical procedures is required to some extent In the Planning Stage As a Substantive Test In the Final Review Stage A. Providing assurance that potential material misstatements will be identified. B. 7. Enhancing the auditor’s understanding of the client’s business and identifying areas of potential risk. D. 9. Searching for unauthorized transactions that may aid in directing unrecorded liabilities. C. Reducing the scope of tests of controls and substantive tests. Obtain evidence from details testing to corroborate particular assertions. B. C. The study of financial ratios is an acceptable alternative to the investigation of unusual fluctuations. Assessing the adequacy of the available evidential matter. Analytical procedures usually are effective and efficient for tests of controls. Plausible relationships among data may reasonably be expected to exist and continue in the absence of known conditions to the contrary. Identify areas that represent specific risks relevant to the audit. Analytical procedures alone may provide the appropriate level of assurance for some assertions. 11. D. D. Relationships involving balance sheet accounts. Analytical procedures used in planning an audit should focus on A. Inquiring about the client’s legal counsel concerning pending litigation. Yes No Yes D. These procedures cannot replace tests of balances and transactions. C. B. Which of the following would not be considered an analytical procedure? A. For audits of financial statements made in accordance with PSAs. Which of the following statements concerning analytical procedures is true? A. Data subject to audit testing in the prior period. B. Developing the expected sales based on the sales trend of the prior five years. Satisfy doubts when questions arise about a client’s ability to continue in existence. No No No 6. Relationships involving income statement accounts. Confirming the existence of the related parties. B. in fact.12. working papers. and reports of internal auditors may often be used as a substitute for the work of the external auditor’s staff. C. 15. occurred an auditor should A. B. Analytical procedures used in the overall review stage of an audit generally include A. Depending on a single product for the success of the entity. B. Efficiency and experience B. 18. D. D. and recommendations. and extent of the external auditor’s procedures. B. When auditing related party transactions. 13. an auditor places primary emphasis on A. reports. The work performed by internal auditors may be a factor in determining the nature. 14. timing. D. B. C. Independence and review skills C. he/she should assess the internal auditors’ A. Gathering evidence concerning account balances that have not changed from the prior year. In assessing the technical competence of an internal auditor. 16. C. Determine whether a particular transaction would have occurred if the parties had not been related. Failing to correct previously identified internal control deficiencies. Writing down obsolete inventory just before year-end. The audit programs. The external auditor should obtain a sufficient understanding of the internal audit function because A. D. Retesting controls that appeared to be ineffective during the assessment of control risk. D. 17. Performing tests of transactions to corroborate management’s financial statement assertions. but were not recorded. and extent of audit procedures. Training and supervisory skills D. Which of the following auditing procedures most likely would assist an auditor in identifying related party transactions. Competence and objectivity 19. Organizational level to which the internal auditor reports. . The understanding of the internal audit function is an important substantive test to be performed by the external auditor. Inspecting correspondence with lawyers for evidence of unreported contingent liabilities. D. Vouching accounting records for recurring transactions recorded just after the balance sheet date. Performing analytical procedures for indications of possible financial difficulties. C. Reviewing confirmations of loans receivable and payable for indications of guarantees. Substantiate that related party transactions were consummated on terms equivalent to those that prevail in arm’s-length transactions. B. Quality of working paper documentation. Which of the following most likely would indicate the existence of related parties? A. an external auditor most likely would obtain information about the A. A. timing. Verifying the valuation of the related party transactions. C. Ascertaining the rights and obligations of the related parties. Considering unusual or unexpected account balances that were not previously identified. C. B. Obtain an understanding of the purpose of the transactions. The procedures performed by the internal audit staff may eliminate the external auditor’s need for considering internal control. Perform analytical procedures to verify whether similar transactions have occurred. Borrowing money at an interest rate significantly below the market rate. Evaluating the disclosure of the related party transactions. If the external auditor decides that it is efficient to consider how the work performed by the internal auditors may affect the nature. After determining that a related party transactions has. Express an unqualified opinion with reference to the work of the expert. Should include in the auditor’s report the identity of the expert and the extent of the expert’s involvement. B. Contracted by the auditor but never by the entity. B. Contracted by the entity but never by the auditor. 23. D. Which of the following is not an expert upon whose work an auditor may rely? A. C. Resolve the matter. The auditor is required to perform substantive test procedures to verify the expert’s assumptions and findings. Contracted but not employed by the entity. 25. Should not refer to the expert’s work. When issuing an unmodified auditor’s report. Appraiser D. Influence of management on the internal auditor’ duties. B. C. Contracted or employed by the entity or auditor. 20. May refer to the work of an expert. scope. D. C. PSA 620 (Using the Work of an Expert) provides guidance on using the work of an expert as audit evidence. C. D. The documentation of this understanding should cover A. The intended use by the auditor of the expert’s work. The auditor should obtain an understanding of the methods and assumptions used by the expert. B. The expert should not have an understanding of the auditor’s corroborative use of the expert’s findings. The should not have an understanding of the nature of the work to be performed by the expert. The auditor’s disclaimer as to whether the expert’s findings corroborate the representations in the financial statements. Report the matter to the appropriate regulatory agency of the government. Withdraw from the engagement. 22. and objective of the expert’s work. C.C. Which of the following statements is correct concerning the auditor’s use of the work of an expert? A. Should refer to the work of an expert to indicate a division of responsibility. the auditor should A. B. A statement that the expert assumes no responsibility to update the expert’s report for future events or circumstances. If the results of the expert’s work do not provide sufficient appropriate audit evidence or are not consistent with other audit evidence. The conditions under which a division of responsibility may be necessary. D.  PSA 500 (revised) Audit Evidence  PSA 501 Audit Evidence – Additional Considerations on Specific Items  PSA 505 External Confirmations  PSA 230 (revised) Audit Documentation 1. an expert may be A. Which of the following statements concerning audit evidence is correct? . Internal auditor 24. D. including the possible communication to third parties of the expert’s identity and extent of involvement. the entity and the expert as to the nature. Actuary B. Entity’s commitment to integrity and ethical values. In using the work of an expert. Engineer C. an understanding should exist among the auditor. 21. D. the auditor A. According to this standard. The measure of the validity of audit evidence lies in the auditor’s judgment. Audit objectives should be developed in light of management assertions about the financial statement components. Existence D. The auditor should resolve any substantial doubt about any of management’s material financial statement assertions. Which of the following statements concerning evidential matter is true? A. Original source documents D. True 3.A. B. B. Financial statements to the potentially unrecorded items. Selection of tests to meet audit objectives should depend upon the understanding of internal control. The difficulty and expense of obtaining audit evidence concerning an account balance is a valid basis for omitting the test. Management makes certain assertions that are embodied in financial statement components. C. Rights and obligations B. D. Supporting evidence to the accounting records. C. In determining whether transactions have been recorded. Potentially unrecorded items to the financial statements. an auditor ordinarily works from the A. D. There should be one. D. Which of the following is not a broad category of management assertions? A. In testing the existence assertion for an asset. Both statements are false. C. The reliability of audit evidence is influenced by its source and by its nature and is dependent on the individual circumstances under which it is obtained. Which of the following is a false statement about audit objectives? A. but it need not be both. C. A client’s accounting records can be sufficient audit evidence to support the financial statements. 7. To be appropriate. True. Completeness C. . False D. Accounting records to the supporting evidence. The quantity of audit evidence needed is affected by the risk of misstatement and also by the quality of such audit evidence. False. 2. General journal entries 8. B. General ledger balances B. The objective of tests of details of transactions performed as substantive tests is to A. audit evidence should be either reliable or relevant. two such categories of assertions are completeness and valuation and allocation. Adjusted trial balance C. B. for example. Evaluate whether management’s policies and procedures operated effectively. Appropriate evidence supporting management’s assertions should be convincing rather than merely persuasive.to –one relationship between audit objectives and procedures. C. D. Comply with generally accepted auditing standards. the direction of the audit testing should be from the A. Attain assurance about the reliability of the accounting system. Detect material misstatements in the financial statements. Both statements are true. Errors or fraud 5. A. 4. 6. B. . B. A. Client representation letter. C. and presentation and disclosures in sufficient detail to form a basis for the assessment of risks of material misstatements and the design and performance of further audit procedures. Bank statements obtained from the client. Assertions about classes of transactions include occurrence. C. Client worksheets supporting cost allocations. 9. Which of the following generalizations does not relate to the appropriateness of evidence? A. 15. Special journals. B. Difficulty and expense involved in testing a particular item. by itself. Which of the following types of audit evidence is the most persuasive? A. PSA 500 requires the auditor to use assertions for classes of transactions. D. An entity’s accounting records can be sufficient audit evidence to support the financial statements. Valuation and and allocation. C. C. A given set of procedures may provide audit evidence that is relevant to certain assertions. 10. Audit evidence obtained from an independent external source is always reliable. 11. B. C. cutoff. An audit usually involves the authentication of documentation. Test counts of inventory made by the auditor. Audit evidence can come in different forms with different degrees of persuasiveness. D. and A. 13. Which of the following is the least persuasive type of evidence? A. Correspondence from the client’s attorney about litigation. 12. Effective internal control contributes little to the reliability of the evidence created within the entity. to be economically useful. The cost of obtaining evidence is not an important consideration to an auditor in deciding what evidence should be obtained. Prenumbered purchase order forms. A client’s accounting records cannot be considered sufficient evidence to support the financial statements. An auditor’s opinion. 14. Audit evidence from external sources (for example. B. Each of the following might. Prenumbered purchase order forms. Assessment of control risk at a low level. Audit evidence generated internally is more reliable when the related accounting and internal control systems are effective. Which of the following statements concerning audit evidence is correct? A. Worksheets supporting cost allocations. Confirmation of accounts receivable. Which of the following is an example of “other information” that could be used by an auditor as evidential matter supporting the financial statements. D. is formed within reasonable time and based on evidence obtained at a reasonable cost.B. D. D. confirmation received from a third party) is more reliable than the generated internally. D. Relationship between the cost of obtaining evidence and its usefulness. Audit evidence obtained directly by the auditor is more reliable than that obtained from the entity. B. but not others. account balances. Accounting manuals. C. completeness. accuracy. Bank statement obtained from the client. Inherent risk involved. D. form a valid basis for an auditor to decide to omit a test except for the A. B. C. Which of the following might be detected by an auditor’s review of the client’s sales cut-off? A. B. D. Completeness and valuation. Comparing receivable turnover ratios with industry statistics for reasonableness. C. D. Assessing the allowance for uncollectible accounts for reasonableness. 18. Unrecorded sales discounts. Existence and completeness . B.B. Excessive goods returned for credit. B. B. C. Vouching amounts in the subsidiary ledger to details on shipping documents. Rights and obligations. Unreturned negative confirmation requests rarely provide significant explicit evidence. C. Which of the following most likely would give the most assurance concerning the valuation and allocation assertion of accounts receivable? A. Negative confirmation requests are effective when detection risk is low. Classification C. Rights and obligations. Valuation and rights and obligations. A substantial number of accounts may be in dispute and the accounts receivable balance arises from sales to many customers with small balances. Rights and obligations C. Inquiring about receivables pledged under loan agreements. Existence B.” Two assertions for which confirmation of accounts receivable balances provides primary evidence are A. B. Cutoff D. Lapping of year-end accounts receivable. D. B. Classification. Cutoff tests designed to detect credit sales made before the end of the year that have been recorded in the subsequent year provide assurance about management’s assertion of A. An auditor most likely would review an entity’s periodic accounting for the numerical sequence of shipping documents and invoices to support management’s financial statement assertion of A. C. Negative confirmation requests are effective when understatements of account balances are suspected. Completeness 19. Inflated sales for the year. Unreturned negative confirmation requests indicate that alternative procedures are necessary. Confirmation is “the process of obtaining and evaluating a direct communication from a third party in response to a request for information about a particular item affecting financial statement assertions. D. 17. Existence 21. 22. C. A small number of accounts may be in dispute and the accounts receivable balance arises from sales to many customers with small balances. In which of the following circumstances would the use of the negative form of accounts receivable confirmation most likely to be justified? A. !6. C. Rights and obligations and existence D. D. Valuation and allocation D. A substantial number of accounts may be in dispute and the accounts receivable balance arises from sales to a few major customers. 20. Existence. Which of the following statements is correct concerning the use of negative confirmation requests? A. A small number of accounts may be in dispute and the accounts receivable balance arises from sales to few major customers. Accuracy of the receivables balance. B. Increase the assessed level of inherent risk for the revenue cycle. When an auditor does not receive replies to positive requests for year-end accounts receivable confirmations. Inventory on consignment and contingent liabilities. the auditor most likely would A. The primary purpose of sending a standard confirmation request to financial institutions with which the client has done business during the year is to A. D. Negative confirmations are to be used for a sample of accounts. Which of the following sets of information does an auditor usually confirm on one form? A. Corroborate information regarding deposit and loan balances. B. Which of the audit procedures is the most appropriate when internal control over cash is weak or when a client request for an investigation of cash transactions? A. Intensify the study of internal control concerning the revenue cycle. C. Cash in bank and collateral for loans. Accuracy of the allowance for bad debts. Inspect the shipping records documenting the merchandise sold to the debtors. D. D. C. Request information about contingent liabilities and secured transactions. B. Review the cash receipts journal for the month prior to year-end. One of the cashiers has been covering a personal embezzlement by lapping. Cash and accounts receivable are audited at the same time. Proof of cash. Which of the following procedures would an auditor most likely to perform for year-end accounts receivable confirmations when the auditor did not receive replies to second requests? A. 26. B. C. D. Increased the assessed level of detection risk for the valuation and completeness assertions. 28. The assessed level of risk of material misstatement relative to financial statement assertions about receivables is acceptably low. D. Bank reconciliation. The credit manager has misappropriated remittances from customers whose accounts have been written off. Increased the assessed level of detection risk for the existence assertion. D. Detect kiting activities that may otherwise not be discovered. B. C. One of the computer control clerks has been removing all sales invoices applicable to his account from the data file. The auditor may consider confirming accounts receivable balances at an interim date if A. 30. The return of positive accounts receivable confirmation without an exception attests to the A. Send the customer a second confirmation. D. C. Evaluate ratio of cash to current liabilities . An auditor confirms a representative number of open accounts receivable as of December 31 and investigates respondents’ exceptions and comments. B. Collections subsequent to year-end are to be reviewed B. 24. 25. B. Accounts payable and purchase commitments. Inspect the allowance account to verify whether the accounts were subsequently written off.23. Provide data necessary to prepare proof of cash. 29. One of the sales clerks has not been preparing charge slips for credit sales to family and friends. C. Accounts receivable and accrued interest receivable. C. By this procedure the auditor would be most likely to learn of which of the following? A. Cash confirmation. Collectibility of the accounts receivable. 27. C. D. Accuracy of the aging of accounts receivable. Items listed in receiving reports and vendors’ invoices to the inventory listing schedule. . B. Which of the following is an audit procedure that an auditor most likely would perform concerning litigation. 32. To gain assurance that all inventory items in a client’s inventory listing schedule are valid. Court records D. The last checks recorded before year-end were actually made by year-end. D. claims. C. Completeness C. Independent auditor 38. claims. B. and assessments is the A. claims. B. Valuation and allocation 36. The primary source of information to be reported about litigation. Sign and return the form without inspecting the accuracy of the client’s bank reconciliation. Which of the following procedures would an auditor most likely perform in auditing the statement of cash flows? A. and assessments indicate a going concern problem. The auditor then traced the test counts to the client’s inventory listing. Be unaware of all the financial relationships that the bank has with the client. 35. an auditor most likely would vouch A. Purchase returns D. Sales returns 37. An auditor selected items for test counts while observing a client’s physical inventory. Client’s management B. The cash receipts journal was held open for a few days after year-end. Items listed in the inventory listing schedule to inventory tags and the auditor’s recorded count sheets. Inventory tags noted during the auditor’s observation to items listed in the inventory listing schedule. While observing a client’s annual physical inventory. 33. The usefulness of the standard bank confirmation request may be limited because the bank employee who completes the form may A. Reconcile the amounts included in the statement of cash flows to the other financial statements’ balances and amounts. Not have access to the client’s cutoff bank statement.31. Inventory tags noted during the auditor’s observation to items listed in the receiving reports and vendors’ invoices. 34. Not believe that the bank is obligated to verify confidential information to third party. D. D. Client’s lawyer C. an auditor recorded test counts for several items and noticed that certain test counts were higher than the recorded quantities in the client’s perpetual records. This procedure most likely obtained evidence concerning management’s assertion of A. Purchase discounts C. This situation could be the result of the client’s failure to record A. B. Vouch all bank transfers for the last week of the year and first week of the subsequent year. Existence D. Cash balances were overstated because of kiting. D. C. C. and assessments? A. An auditor should test bank transfers for the last part of the audit period and first part of the subsequent period to detect whether A. Requests the client’s lawyer to evaluate whether the client’s pending litigation. Sales B. Any unusual payments to or receipts from related parties occurred. Compare the amounts included in the statement of cash flows to similar amounts in the prior year’s statement of cash flows. Reconcile the cutoff bank statements to verify the accuracy of the year-end bank balances. C. Rights and obligations B. Both I and II B. 7 D. Attorneys’ letters D. the auditor should not delete or discard audit documentation before the end of its retention period. and assessment to which the lawyer has devoted substantive attention. 3 B. II only D. claims. D. The permanent (continuing) file of an auditor’s working papers most likely would include copies of the A. the auditor should record I. Pension plan contract D. _____ days after the date of the auditor’s report is ordinarily an appropriate time limit within which to complete the assembly of the final audit file. 5 C.B. Confirm directly with the client’s lawyer that all litigation. II. B. 10 43. As PSQC 1 indicates. Which of the following is not an audit procedure that the independent auditor would perform with respect to litigation. The current file of an auditor’s working papers most likely would include copy of the A. claims. The following are ordinarily excluded from audit documentation: A B C D Superseded drafts of working papers and financial statements Yes No No Yes Notes that reflect incomplete or preliminary thinking Yes Yes No No Previous copies of documents corrected for typographical or other errors Yes Yes Yes Yes Duplicates of documents Yes No Yes No 41. 120 42. 60 B. Lead schedules C. Flowchart of the internal control activities . I only C. and assessments have been recorded or disclosed in the financial statements. The auditor should complete the assembly of the final audit file on a timely basis after the date of the auditor’s report. 90 D. claims. In documenting the nature. Inquire of and discuss with management the policies and procedures adopted for litigation. claims. claims. A. timing and extent of audit procedures performed. and assessments? A. C. Debt agreements 45. Obtain assurance from management that it has disclosed all unasserted claims that the lawyer has advised are probable of assertion and must be disclosed. Neither I nor II 44. Who reviewed the audit work and the date and extent of such review. Articles of incorporation B. C. Discuss with management its policies and procedures adopted for evaluating and accounting for litigation. and assessments that existed at the balance sheet date. Examine the legal documents in the client’s lawyer’s possession concerning litigation. Obtain from management a description and evaluation of litigation. and assertions. D. the retention period for audit engagements ordinarily is no shorter than _____ years from the date of the auditor’s report. 39. After the assembly of the final audit file has been completed. Who performed the audit work and the date such work was completed. and assessments. claims. A. Bank statements B. 40. 30 C. As PSQC 1 indicates. Confirm directly with the client’s lawyer that all claims have been recorded in the financial statements. A. Bank reconciliation C. ”Error” includes A. D. B. Engaging in complex transactions that are structured to misrepresent the financial position or financial performance of the entity. falsification. Intentional misapplication of accounting policies relating to amounts. manner of presentation. I only C. B. III Rationalization. Employee fraud. Management fraud C. Employee fraud. 6. Which of the following conditions are generally present when misstatements due to fraud occur? I Incentive or pressure. Fraudulent financial reporting. C. 3. D. Both I and II B. D. An incorrect accounting estimate arising from oversight or misinterpretation of facts. There are two types of intentional misstatements that are relevant to the auditor: misstatements resulting from fraudulent financial reporting and misstatements resulting from A. I and III only . I and II only. Misstatements in the financial statements can arise from fraud or error. facts that could affect the amounts recorded in the financial statements. A. C. B. 4. Management fraud. C. II and III only. Misappropriation of assets. The distinguishing factor between fraud and error is whether the underlying action that results in the misstatement of the financial statements is I Intentional or unintentional II Rational or irrational A. or not disclosing. Collusion within the entity or with third parties. including A. Concealing. Embezzling receipts. Neither I nor II 2. Stealing physical assets or intellectual property. The auditor is concerned with fraud that causes a material misstatement in the financial statements. II only D. Fraudulent financial reporting involves intentional misstatements including omissions of amounts or disclosures in financial statements to deceive financial statement users. D. Manipulation. It may be accomplished in a number of ways. C. Misappropriation of assets. Using an entity’s assets for personal use. classification. or disclosure.PSA 240 (revised 2005) The Auditor’s Responsibility to Consider Fraud in an Audit of Financial Statements PSA 250 Consider of Laws and Regulations in an Audit of Financial Statements PSA 260 Communications of Audit Matters with those Charged with Governance 1. B. B. or alteration of accounting records or supporting documentation from which the financial statements are prepared. 5. Fraud involving one or more members of management or those charged with governance is referred to as A. II Perceived opportunity. including its internal control. An auditor is responsible to detect material errors but has no responsibility to detect material fraud that is concealed through employee collusion or management override of internal control. Both those charged with governance of the entity and management. When planning the audit. B. When obtaining an understanding of the entity and its environment. Promotions. The following are examples of fraud risk factors relating to misstatements arising from misappropriation of assets. including sales or profitability incentive goals. Adverse relationship between the entity and employees with access to cash or other assets susceptible to theft created by recent changes made to employee compensation or benefit plans. Management of the entity. B. C. Which of the following conditions or events may create incentives/pressures to commit fraud? A. 14. B. The auditor. D. 11. D. an audit of financial statements in accordance with PSAs should be planned and performed with an attitude of A. C. Inadequate system of authorization and approval of transactions. C. Opportunities to misappropriate assets increase when there are A. B. 12. Such events or conditions are referred to as A. Fraud conditions C. C. the auditor may identify events or conditions that indicate an incentive or pressure to commit fraud or provide an opportunity to commit fraud. I. or other rewards inconsistent with expectations. Which of the following statements best describes an auditor’s responsibility regarding misstatements? A. Fraud risk factors. or in high demand. Inadequate segregation of duties or independent checks. B. or fixed assets. D. 13. Lack of mandatory vacations for employees performing key control functions. Inadequate physical safeguards over cash. D. compensation. D. Fraud environment. investments. D. Inventory items that are small in size. B.D. Recurring negative cash flows from operating activities while reporting earnings and earnings growth. D. B. 8. Fraudulent activities. the auditor should make inquiries of management. II. Known or anticipated future employee layoffs. Objective judgment. and III 7. 9. Such inquiries should address the following. Impartial conservatism. of high value. Recent or anticipated changes to employee compensation or benefit plans. An auditor should obtain reasonable assurance that the financial statements taken as a whole are free from material misstatement. The primary responsibility for the prevention and detection of fraud rests with A. C. An auditor’s failure to detect a material misstatement resulting from fraud is an indication of noncompliance with the requirements of the Philippine Standards on Auditing (PSAs). C. Independent integrity. Inadequate access controls over automated records. inventory. Professional skepticism. Because of the risk of material misstatement. Those charged with governance of the entity. whether caused by fraud or error. except A. except . An auditor should obtain absolute assurance that material misstatements in the financial statements will be detected. 10. Excessive pressure on management or operating personnel to meet financial targets established by those charged with governance. B. Management’s consideration of how an element of unpredictability will be incorporated into the nature. Management’s communication. B. If the auditor has obtained evidence that indicates that fraud may exist (even if the potential effect on the financial statements would not be material). 17. the term “noncompliance” as used in the standard refers to acts of omission or commission by the entity being audited. Illegal acts D. B. Which of the following should be documented by the auditor? A. D. either intentional or unintentional. According to PSA 250. and consider the need to report such matters to those charged with governance. Erotic acts B. Transactions entered into on the entity’s behalf by its management or employees. C. Fraud risk factors identified as being present during the auditor’s risk assessment process. which are contrary to the prevailing laws or regulations. Such acts do not include A. this term refers to acts of omission or commission by the entity being audited. Communicate the information to regulatory and enforcement authorities. Noncompliance C. 16. If the auditor has identified a fraud. The following statements relate to communication of misstatements resulting from fraud to management and to those charged with governance.A. Both fraud risk factors identified as being present during the auditor’s risk assessment process and the auditor’s response to any such factors. Withdraw from the engagement. C. The responsibility for the prevention and detection of noncompliance rests with . When the auditor identifies a misstatement in the financial statements. which are contrary to prevailing laws or regulations. C. C. 20. together with the auditor’s conclusion thereon. the auditor should consider whether such a misstatement may be indicative of fraud and if there is such an indication. D. D. to those charged with governance regarding its processes for identifying and responding to the risks of fraud in the entity. B. The auditor need not bring to the attention of those charged with governance any material weaknesses in internal control related to the prevention and detection of fraud. either intentional or unintentional. whether or not it results in a material misstatement in the financial statements. the auditor should A. the auditor should communicate these matters to the appropriate level of management on a timely basis. Auditor’s responses to identified fraud risk factors. Unforgivable acts 19. and extent of the audit procedures to be performed. Management’s process for identifying and responding to the risks of fraud in the entity. A. 15. Transactions entered into by the entity. As used in PSA 250 (Consideration of Laws and Regulations in an Audit of Financial Statements). timing. 18. Personal misconduct (unrelated to the entity’s business activities) by the entity’s management or employees. Management’s assessment of the risk that the financial statements maybe misstated due to fraud. and states that the working papers include the auditor’s reasoning on all significant matters which require the auditor’s judgment. B. Transactions entered into in the name of the entity. and consider the need to report such matters to those charged with governance. The auditor’s communication with those charged with governance may be made orally or in writing. the auditor should communicate these matters to the appropriate level of management on a timely basis. The standard does not require documentation of the identified fraud risk factors and the auditor’s responses to them. if any. Report the matter to the person or persons who made the audit appointment. C. PSA 230 (Documentation) requires the auditor to document matters which are important in providing evidence to support the audit opinion. D. Which is false? A. Consider the implications of the misstatements in relation to other aspects of the audit. D. A qualified or an adverse opinion. Under PSA 260. this term is used to describe the role of persons entrusted with the supervision. An adverse opinion or a disclaimer of opinion. whether or not recorded by the entity that have. An adverse opinion D. C. A qualified opinion or a disclaimer of opinion. Management. Material misstatements in the financial statements. 23. control. are both important and relevant to those charged with governance in overseeing the financial reporting and disclosure process are called A. Which of the following statements relating to communication of audit matters of governance interest is incorrect? A. 29. has. 22. Audit adjustments. the auditor should design audit procedures for the specific purpose of identifying matters of governance interest. the auditor should express A. All of the above. Direction B. . those matters that arise from the audit of financial statements and. The auditor’s communication with those charged with governance may be made orally or in writing. Or could have. If the auditor concludes that the noncompliance has a material effect on the financial statements. the auditor should express A. B.A. The auditor’s lawyer. 28. A qualified opinion or a disclaimer of opinion. D. B. a material effect on its financial statements. and direction of an entity. Audit matters of governance interest to be communicated to those charged with governance ordinarily include A. B. Auditor’s findings D. D. C. In an audit in accordance with PSAs. and has not been properly reflected in the financial statements. C. 26. A qualified or an adverse opinion. The client’s lawyer. C. in the opinion of the auditor. C. Audit matters of governance interest. Significant audit matters. Oversight C. If the auditor is precluded by the entity from obtaining sufficient appropriate audit evidence to evaluate whether noncompliance that may be material to the financial statements. D. B. A. An adverse opinion D. or is likely to have. Material uncertainties related to events and conditions that may cast significant doubt on the entity’s ability to continue as a going concern. The auditor should identify relevant persons who are charged with governance and with whom audit matters of governance interest are to be communicated. Audit matters of governance interest include only those matters that have come to the attention of the auditor as a result of the performance of the audit. occurred. Governance D. 21. An adverse opinion or a disclaimer of opinion 24. 25. Expected modifications to the auditor’s report. Control 27. The auditor. Under PSA 260. C. B. B. III. If the threats are other than clearly insignificant. Professional competence and due care 7. Which of the following independence requirements for assurance engagements is incorrect? A. Evaluate the significance of identified threats to independence. the firm. B. D. . Confidentiality should always be observed by a professional accountant unless specific authority has been given to disclose information or there is a legal or professional duty to disclose. Board of Directors B. C. third parties’ interests that might be affected need not be considered. A. Controller D. A professional accountant’s primary responsibility is to satisfy the needs of an individual client or employer. I and III only. Strict implementation and enforcement of ethical requirements is the best way to prevent unscrupulous acts. Integrity D. The duty of confidentiality continues even after the end of the relationship between the professional accountant and the client or employer. II. Which of the following statements best explains why accountancy profession has found it essential to promulgate ethical standards and to establish means for ensuring their observance? A. In order to achieve the objectives of the accountancy profession. D. For an audit engagement. 5. professional accountants have to observe a number of prerequisites or fundamental principles. The following statements relate to the professional accountant’s obligation to respect the confidentiality of information about a client’s or an employer’s affairs acquired in the course of professional services. Ethical standards that emphasize excellence in performance over material rewards establish reputation for competence and character. B. the members of the assurance team. For corporations covered by the SEC Code of Corporate Governance. I and II only D. Which is false? A. President C. Which of the following fundamental principles requires a professional accountant to be straightforward and honest in performing professional services? A. when the report is not expressly restricted for use by identified users. and network firms are required to be independent of the client. the firm. the members of the assurance team. A professional accountant may disclose confidential information to protect his/her professional interests in legal proceedings. firms. B. Identify threats to independence. PSA 260 requires the auditor to determine the relevant persons who are charged with governance and with whom audit matters of governance interest are communicated. Confidentiality C. II and III. I only C. identify and apply safeguards to eliminate the threats or reduce them to an acceptable level. A distinguishing mark of a profession is acceptance of its responsibility to the public. When authorization to disclose confidential information is given by the client or the employer. and network firms are required to be independent of the client. I. Management CODE OF PROFESSIONAL ETHICS 3. which of the following is primarily responsible for corporate governance? A. B. 14. To ensure that independence of mind and independence in appearance are not compromised. C. For a non-audit assurance engagement. members of the assurance teams.30. Objectivity B. and network firms should I. 15. B. training. The supervision. For a non-audit assurance engagement. misleading or deceptive. B. At all times. be ready to account for the money to any person entitled to such accounting. B. Publicity. Solicitation D. 63. Which of the following is NOT an objective of the Philippine Accountancy Act of 2004? A. Advertising C. . It avoids frequent repetition of. C. The result of the assurance work. D. persons other than management ratify or approve the appointment. D. C. Continuing Professional Education (CPE) requirements. B. It is in good taste and is professionally dignified. Publicity by individual professional accountants in public practice is acceptable provided A. D. when the report is not expressly restricted for use by identified users. and experience requirements for entry into the profession. C. D. For a non-audit assurance engagement. except A. C. All of the above. when the report is expressly restricted for use by identified users. Revised Accountancy Law B. The examination for registration of certified public accountants. C. The communication to the public of facts about a professional accountant which are not designed for the deliberate promotion of that professional accountant is A. The following actions should be taken by a professional accountant in public practice entrusted with clients’ money. Promotion. 2. the members of the assurance team are required to be independent of the client and the firm should not have a material direct or indirect financial interest in the client. B. PHIL ACCOUNTANCY ACT OF 2004 1. Use such money only for the purpose for which it is intended. C. even without the client’s concurrence. except A. Keep such money separately from personal or firm money. The time necessarily occupied by each person engaged in performing the professional services. Which of the following is NOT a factor to consider in determining the professional fee of a professional accountant in public practice? A. and any undue prominence given to the name of the professional accountant in public practice. C. 62. Philippine Accountancy Act of 2004. Educational. Philippine Accountancy Law of 2004. The skill and knowledge required for the type of professional services involved. control. and regulation of the practice of accountancy in the Philippines. Code of Ethics for Professional Accountants. The standardization and regulation of accounting education. legislation or regulation include the following. 23. 60.Republic Act 9298 is known as the A. B. D. When the assurance client’s management appoints the firm. the members of the assurance team and the firm are required to be independent of the client. Safeguards created by the profession. Professional standards and monitoring and disciplinary processes. When it seems likely that the client’s money will remain on client account for a significant period of time. The level of training and experience of the persons necessarily engaged in performing the professional services. D. place such money in an interest-bearing account within a reasonable time. It has as its object the notification to the public of matters of fact in a manner that is not false. 65. No person who has served two successive complete terms as chairman or member shall be eligible for reappointment until the lapse of two (2) years. Practice in Education/ Academe. General average of seventy-five percent (75%). revoke. The members of the Professional Regulatory Board of Accountancy shall be appointed by the A. and practice of accountancy in the Philippines. B. D. licensure. D. Any vacancy occurring within the term of a member shall be filled up for the unexpired portion of the term only. with no grades lower than sixty-five percent (65%) in any given subject. All of the above. with no grades lower than seventy-five percent (75%) in any given subject. To prescribe and adopt the rules and regulations necessary for carrying out the provisions of the Philippine Accountancy Act of 2004. To issue. D. or reinstate the certificate of registration for the practice of the accountancy profession. 10. 3. General average of sixty-five percent (65%). Practice of Public Accountancy B. D. General average of sixty-five percent (65%). All of the above. To supervise the regulation. Association of CPAs in Public Practice (ACPAPP) 9. Appointment to fill up an unexpired term is not to be considered as a complete term. D. Which of the following is not a qualification of an applicant for the CPA licensure examination? A. He/she is of good moral character. President of the Philippines. Philippine Institute of CPAs (PICPA) B. The chairman and members of the BOA shall hold office for a term of three (3). B. He/she is a Filipino citizen. D. The following statements relate to the term of office of the chairman and members of the Board of Accountancy (BOA). To pass the CPA licensure examination. The Board of Accountancy has the power to conduct an oversight into the quality of audits of financial statements through a review of the quality control measures instituted by auditors in order to ensure compliance with the accounting and auditing standards and practices. Which of the following is a function of the Board of Accountancy? A. 15. Peer review D. The following statements relate to the practice of public accountancy. This power of the BOA is called A. He/she is a holder of the degree of Bachelor of Science in Accountancy. Practice in Commerce and Accountancy C. Which is false? A. D. 14. Appraisal B. 6. 20. B. B. C. C. a candidate must obtain a A. Quality review C. General average of seventy-five percent (75%). Quality control. suspend. Which statement is incorrect? . C. The development and improvement of accounting standards that will be generally accepted in the Philippines. 11. Professional Regulation Commission (PRC) C. The practice of accountancy includes A. He/she is at least 21 years of age. C. in accordance with rules and regulations promulgated by the Board and approved by the PRC. PICPA C. D. Single practitioners and partners of partnerships organized for the practice of public accountancy shall be registered CPAs in the Philippines. upon conviction. tribunal. B. C. A foreign CPA called for consultation or for a specific purpose which. Working papers shall be treated confidential and privileged unless such documents are required to be produce through subpoena issued by any court. B. Imprisonment for a period not exceeding two years. B. A fine of not more than P50. that such registrant has acquired a minimum of two(2) years meaningful experience in any of the areas of public practice including taxation. date/place of payment when filed with government authorities or when used professionally. B. A foreign CPA engaged as a professor. be punished by A. The seal of a CPA shall be circular in form. 21. A registered CPA shall obtain and use a seal of a design that will suit his/her taste. B. C. shall. D. in the judgment of the BOA. C. except A. A foreign CPA called for consultation or for a specific purpose which. The seal should be of a design prescribed by the Board bearing the CPA’s name. is essential for the development of the country regardless of whether there are Filipino CPAs who are qualified for such consultation or specific purpose. The Securities Commission shall not register any corporation organized for the practice of public accountancy. Which of the following statements concerning ownership of working papers is incorrect? A.000 or by imprisonment for a period not exceeding two years or both. A partnership engaged in the practice of public accountancy may be carried on in the form of a general partnership (GP) or a limited liability partnership (LLP) organized in accordance with Philippine laws. Which is incorrect? A. is essential for the development of the country and that there is no Filipino CPA qualified for such consultation or specific purpose. 22. A fine of not less than P50. D. AASB .000. in the judgment of the BOA. The following statements relate to the use of seal by registered CPAs. A special/temporary permit may be issued by the Board of Accountancy (BOA) to the following persons. registration number. a Certificate of Accreditation shall be issued to CPAs in public practice only upon showing. 25. Working papers include reports submitted by a CPA to his/her client. All working papers made by a CPA and his/her staff in the course of an examination remain the property of such CPA in the absence of a written agreement between the CPA and the client to the contrary. indicating therein his/her current Professional Tax Receipt (PTR) number. C. Which of the following is the accredited national professional organization of CPAs (APO)? A. and title. D. The auditor’s reports shall be stamped with the CPA’s seal. lecturer or critic in fields essential to accountancy education in the Philippines and his/her engagement is confined to teaching only. A. C. Working papers include schedules and memoranda prepared and submitted by the client to the CPA. D. Any person who shall violate any of the provisions of the Accountancy Act or any of its implementing rules and regulations promulgated by the Board of Accountancy subject to the approval of the PRC. From the effectivity of RA 9298. 23. Lethal injection. A foreign CPA who is an internationally recognized expert or with specialization in any branch of accountancy and his/her service is essential for the advancement of accountancy in the Philippines. or government regulatory or administrative body. 24. C. proficiency and ethical and moral values. Disclosure may be made to any party on consent of the client. B. D. D. .Audit sampling A. When the class of transactions or account balance consists of a large number of small value items. As defined in Annex “C” of the IRR. The first member shall be the president or. The PRC CPE Council shall be composed of a chairperson and two (2) members. FRSC 29. II and III 2. any officer of the organization of deans or department heads of schools. I and III only B. this refers to the inculcation. except A. ASC D. C. Continuing Professional Education. any officer chosen by the Board of Directors of PICPA. The chairperson shall be chosen from among the members of the BOA by the PRC. Selecting all items (100% examination) II. colleges or universities offering the degree requiring licensure examination. It will be appropriate to audit all the items that make up a class of transactions or account balance (100% examination). I. When there is a significant risk of misstatement and other selection methods do not provide sufficient appropriate audit evidence. B. III. The risk of material misstatement related to the assertion being tested. A. C. When the repetitive nature of a calculation or other process performed automatically by the client’s computer information system (CIS) makes a 100% examination effective. Professional Growth and Development 30. The second member shall be the president. the auditor is required to determine appropriate means of selecting items for testing to gather audit evidence. D. Disclosure maybe made to any government agency without subpoena. 33. II only D. Which of the following should be considered by the auditor in deciding which means ( or combination of means) to use in selecting items for testing? I. Both I and II only B. AUDIT SAMPLING PSA 530 1. C. I and II only C. III only D. after the initial registration of a professional that raise and enhance the professional’s technical skills and competence. in his/her absence or incapacity. Audit efficiency. Professional Development. B.In designing audit procedures. Continuing Professional Development B. or in his/her absence or incapacity. Disclosure should not be made even if such disclosure will protect the CPA’s professional interests in legal proceedings. assimilation and acquisition of knowledge. When the class of transactions or account balance consists of a small number of large value items. Which of the following statements concerning a CPA’s disclosure of confidential client information is ordinarily correct? A. Which of the following statements is incorrect concerning the Council’s composition? A. B. skills. Neither I nor II 3. II. Disclosure should be made only if there is a legal or professional duty to make the disclosure. The PRC CPE Council shall assist the BOA in implementing its CPE program. I only C. Which of the following means is/are available to the auditor? I. A. D. Selecting specific items. While performing a test of details during an audit. C. Item in the accounting population be randomly ordered. Assessing control risk too low. Appropriate to the objective of the audit procedure. Does not support the tolerable misstatement for some or all of management’s assertions. This situation illustrates the risk of A. I only C. Population. Can more easily convert the sample into a dual-purpose test useful for substantive testing. as defined in PSA 530. Application of audit procedures to all items that comprise a class of transactions or an account balance. Complete. B. Efficiency of the audit. Effectiveness of the audit. Incorrect acceptance D. No No B. Selection of all items over a certain amount. Audit sampling involves the A. Yes Yes C. not materially misstated. Tolerable misstatement. The likelihood of assessing control risk too high is the risk that the sample selected to test controls A. 10. Contains misstatements that could be material to the financial statements when aggregated with misstatements in other account balances of transactions classes. Neither I nor II 6. An advantage of statistical over nonstatistical sampling methods in tests of controls is that in statistical methods A. D. Eliminate the need to use judgment in determining appropriate sample sizes. Application of audit procedures to less than 100% of items within a class of transactions or an account balance such that all items have a chance of selection. C. Incorrect rejection C. Yes No 11. Stratum of the accounting population be given equal representation in the sample. No Yes D. It was in fact. B. D. means the entire set of data from which a sample is selected and about which the auditor wishes to draw conclusions. 5. Contains proportionately fewer deviations from prescribed internal controls than exist in the balance or class as a whole. B. assessing control risk too high. C.4. B. Does not support the auditor’s planned assessed level of control risk when the true operating effectiveness of internal control justifies such an assessment. 9. II. It is important for the auditor to ensure that the population is I. D. Which of the following sample planning factors would influence the sample size for a substantive test of details for a specific account? Expected Error Tolerable Error A. the auditor determined that the sample results supported the conclusion that the recorded account balance was materially misstated. Application of audit procedures to all items over a certain amount and those that are unusual or have a history of error. B. A. 7. B. Provide an objective basis for quantitatively evaluating sampling risks. C. II only D. D. 8. Afford greater assurance than a nonstatistical sample of equal size. Preliminary estimates of materiality levels. Both I and II only B. An underlying feature of random-based selection of items is that each A. The risk of incorrect acceptance and the likelihood of assessing control risk too low relate to the A. . When performing a test of a control with respect to control over cash receipts. As the population size doubles. Systematic sampling may be appropriate. 17. May occur in a systematic pattern. C. Stop-or-go sampling D. 13. the sample should also double. The documents related to the chosen sample may not be available for inspection. Deviations from control procedures at a given rate usually result in misstatements at a higher rate. an auditor may use a systematic sampling technique with a start at any randomly selected item. Systematic selection C. thus destroying the sample randomness. C. D. Block selection B. If certain forms are not consecutively numbered A. Stratified sampling C. D. C. Increase Decrease Increase B. Attribute sampling 15. Which of the following statistical selection techniques is least desirable for use by an auditor? A. Attributes sampling C. 19. D. The auditor has established a tolerable rate of 5%. Increase Increase Increase . such as peso value? A. Random number tables cannot be used. PPS sampling B. 20. 12. C. Must be systematically replaced in the population after sampling. Item must be systematically selected using replacement. Must be recorded in a systematic pattern before the sample can be drawn. B. Which of the following statements is correct concerning statistical sampling in tests of controls? A. B. B. There is an inverse relationship between the sample size and the tolerable rate. In the review of client invoices the auditor should use A. Increase Increase Decrease D. Selection of a random sample probably is not possible. C. Random-number sampling 16. Which of the following sampling methods would be used to estimate a numerical measurement of a population. D. Sequential sampling D. Must systematically occur more than once in the sample. Variables sampling 18. The biggest disadvantage of this type of sampling is that the items in the population A. An auditor may fail to recognize errors in the documents examined for the chosen sample. Stratified selection D. The quantitative aspects of deviations are not considered by the auditor. Discovery sampling B. An auditor may select audit procedures that are not appropriate to achieve the specific objective. D. The expected population deviation rate of client billing errors is 3%. Variable sampling D. Which of the following combinations results in a decrease in a sample size for attributes? Risk of assessing control risk too low Tolerable rate Expected population deviation rate A. Attributes sampling C. Stratified sampling should be used. Decrease Increase Decrease C. Which of the following statistical sampling plans does not use a fixed sample size for tests of controls? A. B. Sequential selection 14. Item in the accounting population should have an opportunity to be selected. Which of the following best illustrates the concept of sampling risk? A. Variable sampling B. A randomly chosen sample may not be representative of the population as a whole on the characteristic of interest. C. the determination of a sample size sufficient to include at least one such exception is a characteristic of A. C. . Assessing control risk too high. Random sampling B. C. If a random number matches the number of a voided voucher. Variables sampling D. B. There are some observed differences between audited values and book values. B. Identical to the expected rate of errors in the related accounting records. an auditor determined that the sample results supported the conclusion that the recorded account balance was materially misstated. Higher than the expected rate of errors in the related accounting records. Examining invoices in support of the valuation of fixed asset additions. B. 23. It was. Selecting accounts receivable for confirmation of account balances. Consider this test of sample invalid and proceed with substantive tests since internal control can not be relied upon. Increase the sample size to reduce the effect of the unusually large disbursements. For which of the following audit tests would an auditor most likely use attribute sampling? A. D. Treat the missing purchase order as a deviation for the purpose of evaluating the sample. Unrelated to the expected rate of errors in the related accounting records. The total book value is known and corresponds to the sum of all the individual book values. D. An auditor plans to examine a sample of 20 purchase orders for proper approvals as prescribed by the client’s internal control procedures. 22. and the auditor is unable to use alternative procedures to test whether that purchase was properly approved. that voucher ordinarily should be replaced by another voucher in the random sample if the voucher A. 24. 27. This situation illustrates the risk of A. B. The auditor should A. D. C. While performing a test of details during an audit.Use of the ratio estimation sampling technique to estimated peso amounts is inappropriate when A. Which of the following courses of action would an auditor most likely follow in planning a sample of cash disbursements if the auditor is aware of several unusually large cash disbursement? A. Continue to draw new samples until all the unusually large disbursements appear in the sample. C.If the auditor is concerned that a population may contain exceptions. not materially misstated. D. B. The audited values are nearly proportional to the book values. 26.21. Constitutes a deviation. PPS sampling 25. C. Inspecting employee time cards for proper approval by supervisors. B. Incorrect acceptance. An auditor is testing internal control procedures that are evidenced on an entity’s vouchers by matching random numbers. The tolerable rate of deviations for a test of a control is generally A. Discovery sampling C. in fact. A book value for each sample item is unknown. Select a completely new set of 20 purchase orders. Lower than the expected rate of errors in the related accounting records. Assessing control risk too low. D. Incorrect rejection D. Cannot be located. B. Has been properly voided. Set the tolerable rate of deviation at a lower level than originally planned. Making an independent estimate of the amount of FIFO inventory. 28. Stratify the cash disbursements population so that the unusually large disbursements are selected. Represents an immaterial peso amount. D. One of the purchase orders in the chosen sample cannot be found. C. Choose another purchase order to replace the missing purchase order in the sample. A. Related party transactions. Which of the following events most likely indicates the existence of related parties? A. C. B. Borrowing a large sum of money at a variable rate of interest. D. C. Performing tests of transactions to corroborate management’s financial statement assertions. B. D. 4. Securities and Exchange Commission (SEC). This permit the audit of intercompany account balances to be performed as of the concurrent dates. Neither I nor II 5. D. 6. C. The auditor should review information provide by those charged with governance and management identifying I. Retesting controls that appeared to be ineffective during the assessment of control risk. C. II only D. D. B. Entity’s management C. Selling real estate at a price that differs significantly from its book value. Making a loan without scheduled terms of repayment of the funds. Gathering evidence concerning account balances that have not changed from the prior year. Financial Reporting Standards Council (FRSC) D. The communication with the audit committee should be revised. . Internal control activities are not operating effectively. I only C. B. Analytical procedures performed in the overall review stage of an audit suggest that several accounts have unexpected relationships.29. An auditor searching for related party transactions should obtain understanding of each subsidiary’s relationship to the total entity because A. REPRESENTATION LETTER – PSA 580 1. the responsibility for the identification and disclosure of related parties and transactions with such parties rests with the A. This may reveal whether particular transactions would have taken place if the parties had not been related. The results of these procedures most likely indicate that A. Discussing merger terms with a company that is a major competitor.Analytical procedures used in the overall review stage of the audit generally include A. Additional substantive tests of details are required. The names of all known related parties. For which of the following audit COMPLETING THE AUDIT EVENTS AFTER THE BALANCE SHEET DATE – PSA 560 MGT. II. The business structure may be deliberately designed to obscure related party transactions. Auditor B. 2. 3. Considering unusual or unexpected account balances that were not previously identified. Irregularities exist among the relevant account balances. Intercompany transactions may have been consummated on terms of equivalent to arm’s-length transactions. Both I and II only B. D. Confirming a sample of material accounts receivable established after the date of the financial statements. discuss the matter with management. B. 11. unless A. Neither I nor II 9. D. Comparing the financial statements being reported on with those of the prior period. C. and consider taking actions appropriate in the circumstances. The date on which those with the recognized authority assert that they have prepared the entity’s complete set of financial statements. Obtain an understanding of the business purpose of the transaction. Both I and II only B. Investigating personnel changes in the accounting department occurring after the date of the financial statements. Perform analytical procedures to verify whether similar transactions occurred. B. Which of the following statements best expresses the auditor’s responsibility with respect to facts discovered after the date of the audio’s report but before the date the financial statements are issued? A. Information about an event that occurred after the date of the auditor’s report comes to the auditor’s attention. Events occurring between the date of the financial statements and the date of the auditor’s report. occurred. Facts discovered after the date of the auditor’s report. B. the term “subsequent events” refers to I. Add a separate paragraph to the auditor’s report to explain the transaction. Inquiring as to whether any unusual adjustments were made after the date of the financial statements. C. 8. The control environment changes after the issuance of the report. The date that the auditor’s report and audited financial statements are made available to third parties. B. Which of the following procedures would an auditor most likely to perform to obtain evidence about the occurrence of subsequent events? A. but were not recorded. Information. D. As used in PSA 560 (Subsequent Events). After determining that a related party transactions has. If the facts discovered will materially affect the financial statements. The auditor should consider whether the financial statements need amendment. an auditor has no obligation to make continuing inquiries or perform other procedures concerning the audited financial statements. The auditor should amend the financial statements. 12. Which of the following statements best describes the “date of the financial statements”? A. which existed at the report date and may affect the report. The auditor should withdraw from the engagement. II. including the related notes. an auditor should A. which is normally the date of the most recent balance sheet in the financial statements subject to audit. Substantiate that the transaction was consummated on terms equivalent to an arm’s-length transaction. D. comes to the auditor’s attention. D. . After issuing a report. A. and that they have taken responsibility for them. The date on which the auditor has obtained sufficient appropriate audit evidence on which to base the opinion on the financial statements. B. 10. in fact. The date of the end of the latest period covered by the financial statements. Final determinations or resolutions are made of contingencies that had been disclosed in the financial statements. the auditor should issue a new report which contains either a qualified opinion or an adverse opinion. I only C. C. C. II only D.7. C. D. The responsibility to make an assessment of an entity’s ability to continue as a going concern rests with the A. Consider the adequacy of disclosure about the client’s possible inability to continue as a going concern. PSA 570 (Going Concern) states that a fundamental principle in the preparation of financial statements is the going concern assumption. C. Arrearages in principal stock dividends are paid. an entity is ordinarily viewed as continuing in business for the foreseeable future with neither the intention nor the necessity of liquidation. C.13. The discovery of information regarding a contingency that existed before the financial statements were issued. The final resolution of a lawsuit disclosed in the notes to the financial statements. Reconciliation of interest expense with debt outstanding. The entity’s sale of a subsidiary that accounts for 30% of the entity’s consolidated sales. Entity’s management D. A technological development that could affect the entity’s future ability to continue as a going concern. D. 18. Which of the following statements best describes the auditor’s responsibility concerning the appropriateness of the going concern assumption in the preparation of the financial statements? A. Which of the following audit procedures would most likely assist an auditor in identifying conditions and events that may indicate there could be substantial doubt an entity’s ability to continue as a going concern? A. C. Usual trade credit from suppliers is denied. D. . B. Under this assumption. D. Arrearages in principal stock dividends are paid. 20. the auditor’s responsibility is to A. Restrictions on the disposal of principal assets are present. Which of the following events occurring after the issuance of an auditor’s report most likely would cause the auditor to make further inquiries about the previously issued financial statements? A. B. SEC B. When an auditor concludes that there is substantial doubt about a continuing audit client’s ability to continue as a going concern for a reasonable period of time. Review of compliance with terms of debt agreements. B. 17. Which of the following conditions or events most likely would cause an auditor to have substantial doubt about an entity’s ability to continue as a going concern? A. C. Which of the following conditions or events most likely would cause an auditor to have substantial doubt about an entity’s ability to continue as a going concern? A. The auditor’s responsibility is to consider the appropriateness of management’s use of the going concern assumption and consider whether there are material uncertainties about the entity’s ability to continue as a going concern that need to be disclosed in the financial statements. Cash flows from operating activities are negative. Auditor C. B. Entity’s creditors 15. Stock dividends replace annual cash dividends. C. ceasing trading or seeking protection from creditors pursuant to laws and regulations. The auditor’s responsibility is to give a guarantee in the audit report that the entity has the ability to continue as a going concern. D. B. The auditor’s responsibility is to predict future events or conditions that may cause the entity to cease to continue as a going concern. The auditor’s responsibility is to make a specific assessment of the entity’s ability to continue as a going concern. Confirmation of bank balances. Significant related party transactions are pervasive. Confirmation of accounts receivable from major customers. Significant related party transactions are pervasive. 14. 16. should normally be signed by the A. D. Under PSA 580 (Management Representation). Employ analytical procedures as substantive tests to obtain evidence about specific assertions related to account balances. C. Representations by management can be a substitute for other audit evidence that the auditor could reasonably expect to be available. B. Replace a low assessed level of control risk as audit evidence to support the assertion. Chair of the audit committee of the board of directors. Observe the taking of physical inventory on the balance sheet date. reconsider the reliability of other representations by management. and the client’s lawyer. Obtain certain written representations from management. C. C. Chief executive officer and the chief financial officer. B. Chief financial officer and the chair of the board of directors. . when necessary. depending upon materiality. Balance sheet B. acknowledges responsibility for the fair presentation of financial statements. among other matters. 24. which has. C. II. Are not part of the audit evidence considered to support the assertion. When an audit is made in accordance with generally accepted auditing standards. Latest related party transaction C. Reissue the prior year’s auditor’s report and add an emphasis of matter paragraph that specifically refers to “substantial doubt” and “ going concern”. B. 21. Which of the following statements concerning management representations is incorrect? A. A. the auditor is required to obtain audit evidence that management I. B. Acknowledges its responsibility for the fair presentation of the financial statements in accordance with applicable financial reporting framework. Constitute sufficient appropriate audit evidence to support the assertion when considered in combination with a sufficiently low assessed level of control risk. Document the understanding of the client’s internal control and the basis for all conclusions about the assessed level of control risk for financial statement assertions. the auditor should always A. due to the possible effects on the financial statements. Complement. A written representation from a client’s management that. substantive tests designed to support the assertion. Chief executive officer. If a representation by management is contradicted by other audit evidence. C. D. the chair of the board of directors. or may have. Report to the client’s audit committee that management’s accounting estimates may need to be adjusted. a material effect on the financial statements and such audit evidence is expected to be available. an auditor should understand that such representations A. D. I only C. If the auditor is unable to obtain sufficient appropriate audit evidence regarding a matter. even if a representation from management has been received on the matter. Latest interim financial information 26. B. When considering the use of management’s written representations as audit evidence about the completeness assertion. Has approved the financial statements. the auditor should investigate the circumstances and. Neither I nor II 22. 23. Issue a qualified or adverse opinion. D. but do not replace. Auditor’s report D. this will constitute a limitation in the scope of the audit. II only D. 25.The date of the management representation letter should coincide with the date of the A. Both I and II only B. 30. “I believe that the action can be settled for less than the damages claimed. II. I and II only C. An adverse opinion B. Court records B. “I believe that the company will be able to defend this action successfully. Proper grounds to withdraw from the engagement. 32. C. In which of the following circumstances would an auditor most likely meet with the client’s legal counsel to discuss the likely outcome of the litigation and claims? I. Client’s lawyer 29. B. claims and assessments most likely would cause the auditor to request clarification? A. II and II only D. Qualified opinion or a disclaimer of opinion B. Qualified opinion or an adverse opinion. What type of opinion should be expressed if the client’s management refuses to provide a representation that the auditor considers necessary? A. C. Unqualified opinion. B. The letter of audit inquiry should be A. claims and assessments. The auditor determines that the matter is a significant risk. D. The subject matter of the litigation is complex. Adverse opinion or a disclaimer of opinion. Prepared and sent by the auditor. The probable outcome of asserted claims and pending or threatened litigation. An unqualified opinion D. A qualified opinion or a disclaimer of opinion 33. Corroboration of the information furnished by management about litigation. I. Considered to be a scope limitation. D. D. The auditor’s working papers would ordinarily include a summary of oral discussions with management or written representations from management. 27. Prepared by the auditor and sent by management. III. Sufficient reason to issue a “subject to” opinion. C. A qualified opinion or an adverse opinion C. Prepared and sent by management. The primary reason an auditor requests that letters of inquiry be sent to a client’s attorneys is to provide the auditor with A. claims and assessments that existed at the balance sheet date. 28. A description and evaluation of litigation. II and III 34. Management’s refusal to give the auditor permission to communicate with the entity’s legal counsel is most likely to lead to A. I and III only B. Client’s management D.” . D. Which of the following statements extracted from a client’s lawyer’s letter concerning litigation. The primary source of information to be reported about litigation. Prepared by management and sent by the auditor. Insufficient reason to modify the auditor’s report because of the lawyer’s obligation of confidentiality. claims and assessments is the A. A.” B. C. The refusal by the client’s lawyer to provide a representation on the legality of a particular act committed by the client is ordinarily A. There is a disagreement between management and the entity’s legal counsel. 31. D. B. The attorney’s opinions of the client’s historical experiences in recent similar litigation. Independent auditor C. 2. an accountant should A. Date of issuance of financial statements. on a test basis.” D. Provides negative assurance that internal control is functioning as designed. C. D. C. the objective of which is to present financial statements that are free of material misstatements. B.Financial statements of an entity that have been reviewed by an accountant should be accompanied by a report stating that a review A. 4. B. C. Conducted the review in accordance with the Philippine Standard on Review Engagements. D. evidence supporting the amounts and disclosures in the financial statements. “I believe that the possible liability to the company is nominal in amount. REPORTS – OTHER ASSURANCE AND RELATED SERVICES ENGAGEMENTS TO REVIEW FINC’L STATEMENTS 1. B. D. Financial statements of an entity that have been reviewed by an accountant should be accompanied by a report stating that A. When compiling an entity’s financial statements. on a test basis. . Includes examining. Inquire of key personnel concerning related parties and subsequent events. C. B. D. “I believe that the plaintiff’s case against the company is without merit. Date of the auditor’s report. supporting the amounts and disclosures in the financial statements. 5. 3.” 35. Review agreements with financial institutions for restrictions on cash balances. Date of approval of the financial statements. Provides less assurance than an audit. Balance sheet date. on a test basis. The financial statements are the responsibility of the company’s management. an accountant would be least likely to A. The scope of the inquiry and analytical procedures performed by the accountant has not been restricted. Understand the accounting principles and practices of the entity’s industry. Provides only limited assurance that the financial statements are fairly presented. A review includes examining. When compiling the financial statements of an entity. Does not express an opinion or any form of limited assurance on the financial statements. Obtained reasonable assurance about whether the financial statements are free of material misstatements. D. D. Provides only limited assurance that the financial statements are fairly presented. C. An accountant who reviews the financial statements of an entity should issue a report stating that a review A. Examined evidence. Perform analytical procedures designed to identify relationships that appear to be unusual. Perform ratio analyses of the financial data of comparable prior periods.An accountant’s report on a review of the financial statements of an entity should state that the accountant A. B. C. Consists principally of inquiries of company personnel and analytical procedures applied to financial data. information that is the representation of management. Is substantially more in scope than a compilation. Does not contemplate obtaining corroborating evidential matter or applying certain other procedures ordinarily performed during an audit. C. 6. B. The auditor should consider the status of legal matters up to the A. A review is greater in scope than a compilation. D. Plan the work so that an effective engagement will be performed. B. Such prospective financial statements are known as A. No No D. When an accountant examines prospective financial statements. A statement that a compilation of the company’s financial statements was made in accordance with the Philippine Standard on Related Services applicable to compilation engagements. Distribution of the accountant’s report is restricted. Proforma financial statements C. Unlike a projection. C. Negative assurance may be expressed when an accountant is requested to report agreed-upon procedures to specified Elements of a Financial Statements Accounts of a Financial Statement A. The financial statements are prepared in accordance with a comprehensive basis of accounting other than generally accepted accounting principles. A financial forecast consists of prospective financial statements that present an entity’s expected financial position. 10. Disclaims an opinion on whether the assumptions provide a reasonable basis for the prospective financial statements. A forecast A. Present estimates given one or more hypothetical assumptions. An accountant may accept an engagement to apply agreed-upon procedures that are not sufficient to express an opinion on one or more specified accounts or items of a financial statement provided that A. Financial projections D. A statement that management is responsible for the financial statements. The accountant is also the entity’s consulting auditor. B. 7. results of operations. 12. B. an entity’s expected financial position. Partial presentations B. to the best of its knowledge and belief. 8. . Is based on the most conservative estimates. Yes No C. States that the accountant is responsible for events and circumstances up to 1 year after the report’s date. the accountant’s report should include a separate paragraph that A. Obtain an acknowledgement from management of its responsibility for the financial statements. C. Is based on assumptions reflecting conditions expected to exist and courses of action expected to be taken. B. D. C. results of operations. Given one or more hypothetical assumptions. D. Financial forecasts 11. Provides an explanation of the differences between an examination and an audit. D. Which of the following should not be included in an accountant’s report based upon the compilation of an entity’s financial statements? A. Read the compiled financial statements and consider whether they appear to include adequate disclosures. The accountant’s report does not enumerate the procedures performed. a responsible party may prepare. No Yes 9. A statement that the accountant has not audited or reviewed the statements. C. B. C. may contain a range. and cash flows. Contains an opinion as to whether the prospective financial statements are properly prepared on the basis of the assumption and are presented in accordance with generally accepted accounting principles in the Philippines. and cash flows. A statement that the accountant does not express an opinion but provides only negative assurance on the statements. D. Yes Yes B. the auditor’s report should have an appropriate A. the board of directors. Which is false? A. such as by officers of the entity. When in the auditor’s judgment an appropriate level of satisfaction has been obtained. Hypothetical financial information D. including the date of and period covered by the financial statements. A.To distinguish it from reports that might be issued by others. Forecast C. Proforma financial statements B. C. 15. Expresses an opinion on the financial statements. Either to the shareholders or board of directors of the entity whose financial statements are being audited. Which of the following is included in the introductory or opening paragraph of the auditor’s report? A. C. II. A statement that the audit was conducted in accordance with Philippine Standards on Auditing. Before accepting an engagement to examine prospective financial information. Which of the following is a prospective financial information for general use upon which an accountant may appropriately report? A.13. A statement that the responsibility of the auditor is to express an opinion on the financial statements based on the audit. the auditor should consider the intended use of the information. Opinion 2. The auditor should express an opinion as to whether the results shown in the prospective financial information will be achieved. Financial projection C. Identifies the applicable financial reporting framework on which the financial statements are based. 4. The auditor’s report should be addressed A. Only to the board of directors of the entity whose financial statements are being audited. Either to the shareholders or board of directors. the auditor is not precluded from expressing positive assurance regarding the assumptions. B. Title D. or from the reports of other auditors who may not have to abide by the same ethical requirements as the independent auditor. or should withdraw from. Projection B. B. C. Identification of the financial statements audited. of the entity whose financial statements are being audited. 3. Signature B. Best-estimate projection 14. D. or both. A statement that the financial statements re the responsibility of the entity’s management. A prospective financial information prepared on the basis of assumptions as to future events which management expects to take place and the actions management expects to take as of the date the information is prepared (best-estimate assumptions) is known as A. Financial forecast AUDITOR’S REPORT ON FINANCIAL STATEMENTS PSA 700 1. The auditor should not accept. D. Addressee C. B. D. The opinion paragraph of the auditor’s report I. Both I and II only . Only to the shareholders of the entity whose financial statements are being audited. The following statements relate to the examination of prospective financial information. I only C. an engagement to examine prospective financial information when the assumptions are clearly unrealistic. Partial presentation D. II only D. Introductory and Opinion. Implicitly represented in the auditor’s report. D. 6. An auditor who uses the work of an expert may refer to and identify the expert in the auditor’s report if the A. Unqualified opinion C. Neither I nor II 5. Explicitly represented in the “Auditor’s Responsibility” paragraph of the auditor’s report.e. Adverse opinion B. . Management’s Responsibility and Opinion. Disclaimer of opinion D. B. Expert is employed by the entity. Which paragraphs of an auditor’s report on financial statements should refer to Philippine Financial Reporting Standards? A. D. Auditor indicates a division of responsibility related to the work of the expert. C. The following statements relate to the date of the auditor’s report. D. Expert’s work provides the auditor greater assurance of reliability. B. 9. A note to the financial statements of the Prudent Bank indicates that all of the records relating to the bank’s business operations are stored on magnetic disks. Which is false? A. the balance sheet date). 8. The auditor should date the report as of the completion date of the audit. 7. C. The company’s total assets and income before tax are P70 million and P15 million. the auditor’s report should express A. In which of the following circumstances would an auditor most likely add an emphasis of matter paragraph to the auditor’s report while expressing an unqualified opinion? A. The date of the auditor’s report should not be later than the date on which the financial statements are signed or approved by management. An auditor’s responsibility to express an opinion on the financial statements is A. A “subject to” opinion 11. B. D. Assuming no other issues affect the report. Explicitly represented in the “Management’s Responsibility” paragraph of the auditor’s report. the auditor’s report will most likely contain a/an A. A qualified opinion C. Certain transactions cannot be tested because of management’s records retention policy. An unqualified opinion D. Management’s estimates of the effects of future events are unreasonable. and that no emergency backup systems or duplicate disks are stored because the bank and its auditors consider the occurrence of a catastrophe to be remote.000. D. respectively. B. An adverse opinion is expressed based on a difference of opinion between the client and the outside appraiser as to the value of certain assets. B. No depreciation has been provided in the financial statements. Auditor’s Responsibility and Management’s Responsibility C. Based upon this note. 12. Introductory and Auditor’s Responsibility. An adverse opinion B. When would an auditor refer to the work of an appraiser in the auditor’s report? A. The date of the auditor’s report should always be later than the date on which the financial statements (i. B. C. Auditor expresses a qualified opinion or an adverse opinion related to the work of the expert.. The date of the auditor’s report should not be earlier than the date on which the financial statements are signed or approved by management. Scope qualification 10. An independent auditor discovers that a payroll supervisor of the company being audited has misappropriated P50. There is substantial doubt about the entity’s ability to continue as a going concern. C. Explicitly represented in the opinion paragraph of the auditor’s report. An unqualified opinion B. When two or more auditing firms participate in an audit. C. If the principal auditor makes reference to another auditor in an audit that would otherwise result in an unqualified opinion. on reading the other information. No Yes C. Disclaimer of opinion. Yes No D. one firm should be the principal auditor. Disclaim an opinion on the financial statements after explaining the material inconsistency n an emphasis of matter paragraph. C. D. Han an obligation to perform auditing procedures to corroborate the other information. D. A disclaimer of opinion is expressed because of a scope limitation imposed on the auditor by the appraiser. Which of the following should be included in the opinion paragraph when an auditor expresses a qualified opinion? When Read in Conjunction with Note X With the Foregoing Explanation . An adverse opinion 16. A qualified opinion D. Qualified or adverse opinion. D. the type of audit report issued should be A. A qualified opinion is expressed because of a matter unrelated to the work of the appraiser. Qualified opinion or disclaimer of opinion. No No B. B. Yes Yes 20. but the client refuses to revise or eliminate the material inconsistency. When audited financial statements are presented in a document (e. the auditor identifies a material inconsistency. 18. Consider the matter closed because the other information is not in the audited statements. C. “If. B. 19. Is required to express a qualified opinion if the other information has a material misstatement of fact. A disclaimer of opinion C. PSA 720 states.” What type of opinion should be expressed if the client refuses to make the necessary amendment in the financial statements? A.. An unqualified opinion is expressed and an emphasis of matter paragraph is added to disclose the use of the appraiser’s work. B. 17.g. B. If the auditor concludes that the financial statements do not require revision. D. C. the auditor may A. the auditor should determine whether the audited financial statements or the other information needs to be amended. 13. the auditor A. An auditor concludes that there is a material inconsistency in the other information in an annual report to shareholders containing audited financial statements. Should read the other information to consider whether it is inconsistent with the audited financial statements. An auditor may express a qualified opinion under which of the following circumstances? Lack of Sufficient Appropriate Evidence Restriction on the Scope of the Audit A. Has no responsibility for the other information because it is not part of the basic financial statements. annual report) containing other information. Revise the auditor’s report to include an emphasis of matter paragraph describing the material inconsistency. Unqualified opinion with an emphasis of matter paragraph describing the material inconsistency. Express a qualified opinion after discussing the matter with the client’s directors. D. Unqualified opinion D. the corresponding figures for the prior period(s) are integral part of the current period financial statements. With the foregoing explanation of these omitted procedures. Subject to the departure from generally accepted accounting principles. Under the corresponding figures framework. II. the corresponding figures for the prior period(s) are considered separate financial statements. The financial statements fail to disclose information that is required by Philippine Financial Reporting Standards. Under the corresponding figures framework. B. Which of the following phrases would an auditor most likely include in the auditor’s report when expressing a qualified opinion because of inadequate disclosure? A. There are two broad financial reporting frameworks for comparatives: the corresponding figures and the comparative financial statements. 23. the auditor’s report only refers to the financial statements of the current period. Events disclosed in the financial statements cause the auditor to have substantial doubt about the entity’s ability to continue as a going concern. C. Yes No B. and of its financial performance and its cash flows for the year then ended in accordance with Philippine Financial Reporting Standards. An auditor’s report includes the following statement: “In our opinion. Yes Yes 21. D. 25. Under the corresponding figures framework. 22. No Yes C. C. Which of the following statements is correct concerning these reporting frameworks? A. Do not present fairly in all material respects. B. No No D. A. The following statements relate to the auditor’s reporting responsibilities regarding comparatives. Under the corresponding figures framework. The auditor did not observe the entity’s physical inventory and is unable to become satisfied as to its balance by other auditing procedures.For comparative financial statements.” This auditor’s report contains a/an A. 26. Within the notes to the financial statements. Following the opinion paragraph B. C. Except for the omission of the information included in the preceding paragraph. the comparative financial statements figures for the prior period(s) are intended to be read in conjunction with the amounts and other disclosures relating to the current period. the amounts and other disclosures for the prior period(s) form part of the current period financial statements. In which of the following situations would an auditor ordinarily choose between expressing a qualified opinion or an adverse opinion? A. An auditor should disclose the substantive reasons for expressing an adverse opinion in an emphasis of matter paragraph A. Qualified opinion C. The auditor wishes to emphasize an unusually important subsequent event. because of the effects of the matters discussed in the preceding paragraph. For corresponding figures. in all material respects. the financial statements do not present fairly. Following the introductory paragraph D. the financial position of ABC Company as of December 31. 20X1. Disclaimer of opinion B. the auditor’s report refers to each period that financial . B. as described above. D. Which is incorrect? I. Preceding the opinion paragraph C. Adverse opinion 24. audited MDG Company’s prior-year financial statements. A. II. The auditor’s procedures are ordinarily limited to ensuring that the corresponding figures have been correctly reported and are appropriately classified. II and III only. B. D. and the corresponding figures have not been properly restated and/or appropriate disclosures have not been made. B. which expressed a qualified opinion. When the prior period financial statements are not audited. D. 30. Accounting policies used for the corresponding figures are consistent with those of the current period or whether appropriate adjustments and/or disclosures have been made. C. A. the incoming auditor should state in the auditor’s report that I. When the auditor’s report on the prior period. II only. The corresponding figures are unaudited. II only D. I and III only. A. I only. included a modified opinion and the matter which gave rise to the modification is resolved and properly dealt with in the financial statements. II. That the financial statements of the prior period were audited by another auditor. I and II only. as previously issued. The incoming auditor is not required to perform procedures regarding opening balances of the current period. These statements are presented with those of the current year for comparative purposes without Ruth’s auditor’s report. CPA. II only D.Correponding figures agree with the amounts and other disclosures presented in the prior period or whether appropriate adjustments and/or disclosures have been made. C. III. When the auditor’s report on the prior period financial statements containing a material misstatement included an unmodified opinion and the prior period financial statements have not been revised and reissued. the incoming auditor may refer to the predecessor auditor’s report on the corresponding figures in the incoming auditor’s report for the current period. In which of the following circumstances would an auditor’s report least likely include specific reference to the corresponding figure? A. D. In drafting the currents year’s auditor’s report. should . I only C. The type of report issued by the predecessor auditor. When the auditor’s report on the prior period. Both I and II. included a modified opinion and the matter which gave rise to the modification is unresolved. the incoming auditor. B. The incoming auditor’s report should indicate I. CPA. II. A. as previously issued. Neither I nor II 28. Christine Marie. and results in a modification of the auditor’s report regarding the current period figures. as previously issued. 29. According to PSA 710. included a modified opinion and the matter which gave rise to the modification is unresolved but does not result in a modification of the auditor’s report regarding the current period figures. C. Both I and II only B. 31. II and III. I only C. I. Mary Ruth.The date of the predecessor auditor’s report. Both I and II only B. When the auditor’s report on the prior period. statements are presented. Neither I nor II. The auditor should assess whether I. Neither I nor II 27. PSA 710 states that the extent of audit procedures performed on the corresponding figures is significantly less than for the audit of the current period figures. When comparative financial statements are presented. III and IV. who is satisfied after properly communicating with the incoming auditor. the auditor’s opinion on the financial statements “taken as a whole” should be considered to apply to the financial statements of the A. D. II. IV. I. D. Indicate the type of report issued by Mary Ruth. II and II only. B. II and IV only. I. III. The incoming auditor should state in the auditor’s report that the comparative financial statements are unaudited. D. I. . 34. Request the client to reissue the predecessor’s report on the prior year’s statements. A. The following statements relate to unaudited prior year financial statements that are presented in comparative form with audited current year financial statements. C. Not name Mary Ruth as the predecessor auditor. Current period and those of the other periods presented. The predecessor auditor’s report should A. the incoming auditor should A. Issue an updated comparative audit report indicating the division of responsibility. Not refer to the report or the work of the incoming auditor. D. C. C. B. Refer to the work of the incoming auditor in the scope and opinion paragraphs. The incoming auditor need not perform audit procedures regarding opening balances of the current period. Current and immediately preceding period only. Periods presented plus one preceding period. C. B. If the predecessor’s report was qualified. The predecessor auditor. 32. II. Clear disclosure in the financial statements that the comparative financial statements are unaudited is encouraged. B. has reissued his/her auditor’s report on prior year financial statements. Which is incorrect? A. III and IV only. the auditor should request management to revise the prior year’s figures or if management refuses to do so. In situations where the incoming auditor identifies that the prior year unaudited figures are materially misstated. II. appropriately modify the report. D. Indicate the substantive reasons for the qualification in the predecessor auditor’s opinion. B.Indicate the date of Mary Ruth’s auditor’s report. 35. Comparative financial statements include the financial statements of the prior year that were audited by a predecessor auditor whose report is not presented. Current period only. 33. Refer to the report of the incoming auditor only in the scope paragraph. I. C. Express an opinion only on the current year’s statements and make no reference to the prior year’s statements. Refer to both the work and the report of the incoming auditor only in the opinion paragraph.Indicate the substantive reasons for Mary Ruth’s qualification.
Copyright © 2024 DOKUMEN.SITE Inc.